18.07.2024

Определить общее сопротивление: Как вычислить общее сопротивление цепи

Содержание

Как найти общее сопротивление цепи формула. Последовательное соединение резисторов

Бондаров Михаил Николаевич

Учитель физики лицея №1501 и ГОУ ЦО «Технологии обучения» г. Москвы.

Расч¸т сопротивления электрической цепи

В статье на примерах решения конкретных задач показано применение различных способов расч¸та сопротивления электриче- ской цепи. Рассмотрены некоторые методы преобразования электрических схем к эквивалентным схемам.

Введение

Задачи на расч¸т сопротивления электрической цепи можно встретить среди экзаменационных и олимпиадных задач, а также в заданиях ЕГЭ. В данной статье мы рас-

смотрим некоторые при¸мы, позволяющие достаточно эффективно определять сопротивления различных цепей. Напомним основные закономерности.

1. Последовательное и параллельное соединения проводников

В электрических цепях, с которыми можно встретиться на практике, проводники соединяются различными способами. Наиболее простые соединения известны как последовательное и параллельное соединения.

При последовательном соединении один проводник следует за другим так, что конец одного служит началом другого (рис. 1). Обратите внимание, что при этом между проводниками нет разветвлений проводов.

Из курса физики 8 класса известно, что общее сопротивление R последовательно соедин¸нных про-

водников сопротивлениями R
1
èR
2

определяется по формуле:

R
1R
2.

При параллельном соединении проводники подключаются к одной и той же паре точек А и В (рис. 2).

Общее сопротивление R парал-

соедин¸нных

проводников

сопротивлениями R
1
èR
2
опреде-

ляется из соотношения:

2. Распознавание типов соединений

Для уверенного

соединены последовательно, не за-

на определение общего сопротивле-

мечая того, что узлы C и D между

электрической

ними свидетельствуют о разветвле-

уметь в сложных схемах распозна-

нии проводов.

вать проводники, соедин¸нные по-

Рассмотрим

следовательно или параллельно.

одну схему (рис. 4). Попробуйте

Рассмотрим конкретные примеры.

найти в ней последовательно и

Пример 1. В схеме, изображ¸н-

параллельно

соедин¸нные

на рис. 3, только проводники

водники (свой ответ проверьте по

R2
,

R
4
èR
6
соединены последо-

вательно. Иногда

ошибочно

тают, что все проводники, кроме R
3
,

3. Смешанное соединение проводников

Приступим теперь к расч¸ту сопротивления при смешанном соединении проводников. Начн¸м с простых цепей.

Задача 1. Определите общее сопротивление цепи, схема которой изображена на рис. 3, если все проводники имеют сопротивление по 4 Ом.

Решение. Учитывая анализ, провед¸нный в примере 1, выделим три последовательно соедин¸нных проводника R
2
,R
4
èR
6
(ðèñ. 5).

На эквивалентной схеме (рис. 6) их можно заменить одним проводником сопротивлением

R
246R
2R
4R
612 Îì.

Теперь ясно видно, что проводник сопротивлением R
246
соедин¸н

параллельно с проводником сопротивлением

R
3.

Определим их общее сопротивление
R
2463:

R
2463

R
246

R
2463

R
246

R
246

На эквивалентной схеме (рис. 7) видно, что исчезли узлы между

проводниками R
1
,R
2463
èR
5
, поэтому эти проводники оказались соедин¸нными последовательно, и их общее сопротивление

R R
1R
2463R
511 Îì.

Ответ: 11 Ом.

Задача 2. В цепи, схема которой изображена на рис. 8, все проводни-

ки имеют сопротивление по 10 Ом. Чему равно общее сопротивление цепи между точками А и В?

Решение. Обратите внимание, что точки В и D соединены проводом с нулевым сопротивлением, следовательно, их можно объединить в один узел. Тогда эквивалентная схема будет иметь вид, изобра- ж¸нный на рис. 9.

Теперь общее сопротивление цепи легко находится в три этапа.

1) Сначала вычисляем сопротивление параллельно соедин¸нных проводников
R

R
4:

На преобразованной схеме вместо проводников R
2
èR
4
рисуем эквивалентный им проводникR
24
(ðèñ. 10 a).

2) Затем определяем сопротивление последовательно соедин¸нных проводников R
24
è R1
:

R
241R
24R
115 Îì.

На рис. 10 б показана новая эквивалентная схема.

3) Наконец, находим общее сопротивление цепи:

Ответ: 6 Ом.

Заметим, что преобразуя цепь к более простой эквивалентной, можно на схемах изгибать, удлинять или укорачивать провода, перемещать узлы вдоль проводов, поскольку провода, соединяющие элементы схемы, считаются идеальными, т. е. имеющими нулевое сопротивление.

Рассмотрим теперь, как может видоизмениться цепь, состоящая из тр¸х последовательно соедин¸нных проводников, если к ним добавить другие проводники. Сначала добавим два проводника с нулевым сопротивлением, соединив ими точку А с точ- кой С, а В – с D (рис. 11).

Задача 3. Найдите сопротивление между точками А и D цепи (рис. 11), если каждый проводник имеет сопротивление 3 Ом.

Решение. Так как точки А и С соединены проводом с нулевым сопротивлением, то их потенциалы равны, а значит, эти точки имеют одинаковый потенциал, и их можно собрать в один общий узел. Аналогично поступим с точками В и D. Таким образом, полу- чилось, что каждый проводник оказался подключенным к одним и тем же точкам, т. е. все три проводника соединены параллельно. Теперь уже легко найти общее сопротивление цепи, учи- тывая, что все проводники имеют одинаковое сопротивление:

Ответ: 1 Ом.

Заменим теперь один из идеальных проводников реальным – с ненулевым сопротивлением.

Задача 4. Определите сопротивление между точками А и D цепи (рис. 12), если каждый проводник имеет сопротивление 6 Ом.

Заметим, что точки В и D попрежнему соединены проводником с нулевым сопротивлением. Следовательно, они имеют одинаковый потенциал. Соединив их в один узел, получим эквивалентную схему (рис. 13 а). Если же затем «распрямить» е¸, то участки с параллельно и последовательно соедин¸н- ными проводниками станут хорошо видны (рис. 13 б).

После этого останется лишь произвести расч¸т сопротивления для эквивалентной схемы в три этапа:

R R
1R
234

R
234

R
234

R
234

Ответ: 3,6 Ом.

4. Мостиковая схема

Заменив второй идеальный проводник в цепи, схема которой показана на рис. 11, реальным, получим новую цепь, схема которой изображена на рис. 14.

Задача 5. Определите сопротивление между точками А и D цепи, схема которой изображена на рис. 14, если сопротивление каждого проводника равно 2 Ом.

Решение. На этот раз все попытки обнаружить хотя бы одну пару последовательно или параллельно соедин¸нных проводников оказываются безуспешными. И вс¸ же попробуем немного видоизменить схему так, чтобы можно было увидеть в ней элементы симметрии. Немного потренировавшись, можно получить следующую схему – она носит название «мостиковая схема» (рис. 15).

Поскольку все проводники имеют одинаковое сопротивление, то по

ветвям ABD и ACD текут одинаковые токи, а через выделенную красным цветом перемычку ВС ток идти не будет. Поэтому перемычку можно безболезненно удалить из цепи, не изменив при этом общее сопротивление цепи (рис. 16).

Заметим кстати, что ток через перемычку не будет идти при любом е¸ сопротивлении
, если для остальных проводников выполняется соотношение:

Теперь же ясно видно, что цепь состоит из двух параллельно соеди- н¸нных участков, в которых находится по два последовательно соеди- н¸нных проводника. Определим сна- чала сопротивление каждого из этих участков: R
ABD
R
AСD
(2 2) Ом

4 Ом. Затем находим общее сопротивление цепи:

Ответ: 2 Ом.

Задача 6. Каково сопротивление

между точками N и M цепи, схема которой показана на рис. 17, если сопротивления проводников

R
1
1 Ом,R
2
2 Ом,R
3
4 Ом,R
4
2 Ом,R
5
5 Ом?

Решение. В данном случае не очевидно, что ток через выделенный красным цветом проводник 5 идти не будет. И вс¸ же, проделав несколько манипуляций с цепью, можно полу- чить мостиковую схему (этапы преобразований показаны на рис. 18 a–ã).

Обратим внимание, что по условию задачи

R
1R
2,

R
4R
3

то есть выполняется соотношение (*). Следовательно, потенциалы точек С и D одинаковы, через проводник 5 ток не ид¸т, и его можно удалить, не изменяя сопротивление цепи. После этого эквивалентная схема выглядит совсем просто (рис. 18 д).

Произвед¸м

сопротивле-

íèÿ öåïè:

Ответ: 2 Ом.

Заметим, что если все попытки

распутать и упростить схему из пя-

ти проводников не приводят к успе-

ху, вполне вероятно, что вы натолк-

нулись на «мостиковую схему» (как,

например, на рис. 17).

При определении общего сопро-

тивления в задачах 5 и 6 мы ис-

пользовали симметрию схемы. По-

знакомимся более подробно с при¸-

мами, позволяющими рассчитывать

5. Метод исключения участков цепи

Задача 7. Определите сопротивление между точками А и С цепи (рис. 19), если каждый проводник имеет сопротивление 3 Ом.

Решение. Из симметрии схемы (рис. 20) следует, что по выделенным зел¸ным цветом проводникам будут идти одинаковые токи, следовательно, потенциалы точек В, О и D будут одинаковы. Тогда по проводникам, выделенным красным цветом, токи идти не будут, и их можно удалить из цепи, не изменив при этом е¸ общее сопротивление.

В результате схема упростится (рис. 21), выделенные одинаково проводники тр¸х ветвей окажутся соедин¸нными последовательно, а сами ветви соединены между собой параллельно.

Теперь уже дальнейший расч¸т несложен: сопротивление каждой ветви равно 6 Ом, а общее сопротивление цепи составляет 2 Ом.

Ответ: 2 Ом.

Заметим, что именно этот при¸м был использован при решении задач 5 и 6.

получим более простую схему (рис. 24).

Заменим теперь параллельно соедин¸нные проводники 2 и 56 проводником с эквивалентным сопротивлением R
256
:

R
256

R
256

R
2R
56

R
2R
56

Поступим аналогично и с проводниками 78 и 4: эквивалентное сопротивление R
784
также равно 4 Ом.

Теперь схема ещ¸ более упрощается (рис. 25).

Дальнейший расч¸т несложен:

7. Метод склеивания узлов

Этот метод является обратным по отношению к предыдущему.

Задача 9. В каждое из р¸бер куба включ¸н проводник сопротивлением 6 Ом. Чему равно сопротивление получившейся конструкции между вершинами А и В, находящимися на концах большой диагонали куба?

Решение. Используем теперь метод склеивания узлов.

Изобразим на чертеже схему включения проводников (рис. 26). Учитывая симметрию схемы, заметим, что токи через проводники, выделенные зел¸ным, одинаковы. Следовательно, потенциалы точек 1, 2 и 3 равны. Тогда, если их соединить идеальными проводниками, то по этим проводникам ток идти не будет, а значит общее сопротивление цепи не изменится. Таким образом, точки 1, 2 и 3 оказались собранными в один узел, а выделенные зел¸ным проводники стали соедин¸нными параллельно.

Поступив аналогично с проводниками, выделенными синим, полу- чим три других параллельно соеди- н¸нных проводника.

Обратим внимание, что шесть оставшихся проводников (они выделены ж¸лтым цветом) теперь оказываются соедин¸нными параллельно. Действительно, каждый из них одним концом подключ¸н к узлу, выделенному зел¸ным цветом, а другим – к узлу, выделенному синим цветом.

Таким образом, эквивалентная схема состоит из тр¸х последовательно соедин¸нных участков: 1) три параллельно соедин¸нных проводника; 2) затем ещ¸ шесть; 3) и, наконец, снова три (рис. 27).

Аналогично произвед¸нному в задаче 3 расч¸ту сопротивление тр¸х параллельно соедин¸нных проводников равно 2 Ом, а у шести таких же проводников – вдвое меньше: 1 Ом. Теперь эквивалентная схема состоит из тр¸х последовательно соедин¸нных проводников, и их общее сопротивление

R
(2 1 2) Ом 5 Ом.

Ответ: 5 Ом.

Заключение

Итак, подвед¸м краткий итог. При определении общего сопро-

1. Метод склеивания узлов. Если два или более узлов имеют одинаковый потенциал, то их можно соединить в один узел.

2.
Метод исключения участков цепи. Проводник можно удалить, если через него не теч¸т ток (узлы, которые он соединяет, имеют одинаковый потенциал).

3.
Метод разрезания узлов. Действие, противоположное склеиванию узлов.

А что делать в том случае, если число проводников в цепи стремится

ê
бесконечности или не уда¸тся найти не только последовательно или параллельно соедин¸нные проводники, но и симметрию в схеме? Об этом будет рассказано в другой статье.

Ответ на пример 2

Ж¸лтым цветом выделены последовательно соедин¸нные проводники, зел¸ным – параллельно соедин¸нные.

Задачи для самостоятельного решения

R
12563

R
1R
256R
316 Îì;

1. В цепи, схема которой изо-

рой показана на рис. 19, если каж-

бражена на рисунке 29, все провод-

дый проводник имеет сопротивление

ники имеют сопротивление 8 Ом.

15 Ом. (Ответ: 7 Ом.)

Определите общее сопротивление

3. В каждое из р¸бер куба вклю-

цепи. (Ответ: 13 Ом.)

чен проводник сопротивлением 8 Ом.

Чему равно сопротивление полу-

чившейся конструкции между вер-

шинами, принадлежащими одной из

граней и лежащими на концах е¸

диагонали (на рис. 26 это вершины

А и 6)? (Ответ: 6 Ом.)

4. Определите сопротивление ме-

жду точками А и В цепи, схема ко-

2. Определите сопротивление ме-

торой изображена на рис. 30, если

жду точкам А и О цепи, схема кото-

Если не использовать специальные технические решения по увеличению проводимости сопротивление имеют все проводники электротока. Даже провод может оказаться не пригодным при большой силе тока и его придётся заменять, например, медной шиной. Определить величины тока в источнике питания, и в нагрузках позволяет расчет сопротивления электрической цепи .

Два вида соединений элементов может быть в электрической схеме:

  • последовательное;
  • параллельное.

На изображении выше показаны четыре нагрузки. Из них R1 и R2 соединены последовательно, так же, как и нагрузка R3 c R4. Но в этой схеме есть и параллельное соединение: R1и R2 параллельны R3 и R4.

В любых электрических цепях элемент с функциями сопротивления именуется как «резистор». Если в электрической схеме n
резисторов соединены последовательно величины их сопротивлений суммируются. Следовательно, общее сопротивление электрической цепи Rобщ.

При параллельном соединении n
резисторов для Rобщ. справедливо выражение

Сколько бы ни было резисторов в электрической схеме их можно заменить одним элементом эквивалентного сопротивления и определить ток в этом элементе по закону Ома. Для этого шаг за шагом выполняется замена нескольких резисторов одним в соответствии с их соединением – параллельным или последовательным. На примере первого изображения с резисторами R1, R2, R3 и R4 получится следующее. Сначала заменяем R1 и R2 на R5:

Затем заменяем R3 и R4 на R6:

В результате получается новая эквивалентная схема из R5 и R6, соединённых параллельно

А конечный результат определить легко:

  • 1/Rобщ.=1/R5 + 1/R6,
  • Rобщ.=
    R5*R6/(R5+R6)

В том случае, когда не удаётся сразу сделать замену нескольких элементов одним, как например, на схеме, показанной ниже

следует сделать её анализ. В приведенном изображении, очевидно, что сумма токов в нагрузках R1 и R2 равна сумме токов в нагрузках R4 и R5. Следовательно, потенциалы точек А и В равны, и могут быть объединены. Ток через нагрузку R3 не течёт. Получаются такие соединения

Эти соединения нагрузок легко преобразуется сначала в два последовательно соединённых элемента, а затем в одно эквивалентное сопротивление. Для более сложных и разветвлённых цепей применяется такой же метод.

Последовательное соединение
это соединение двух или более резисторов в форме цепи, в которой каждый отдельный резистор соединяется с другим отдельным резистором только в одной точке.

Параллельное соединение
это соединение, при котором резисторы соединяются между собой обоими контактами. В результате к одной точке (электрическому узлу) может быть присоединено несколько резисторов.

2) Общее сопротивление R общ

При таком соединении, через все резисторы проходит один и тот же электрический ток. Чем больше элементов на данном участке электрической цепи, тем «труднее» току протекать через него. Следовательно, при последовательном соединении резисторов их общее сопротивление увеличивается, и оно равно сумме всех сопротивлений.

Общее сопротивление R общ

При таком соединении, через каждый резистор потечет отдельный ток. Сила данного тока будет обратно пропорциональна сопротивлению резистора. В результате общая проводимость такого участка электрической цепи увеличивается, а общее сопротивление в свою очередь уменьшается.

Таким образом, при параллельном подсоединении резисторов с разным сопротивлением, общее сопротивление будет всегда меньше значения самого маленького отдельного резистора.

Формула эквивалентного общего сопротивления при параллельном соединении резисторов:

Для двух одинаковых резисторов общее сопротивление будет равно половине одного отдельного резистора:

Соответственно, для n одинаковых резисторов общее сопротивление будет равно значению одного резистора, разделенного на n.

3)Электропроводность, электрическая проводимость, проводимость, способность тела пропускать электрический ток под воздействием электрического поля, а также физическая величина, количественно характеризующая эту способность. Тела, проводящие электрический ток, называются проводниками, в отличие от изоляторов.. .
Основная единица измерения сопротивления — Ом. Удельная проводимость — величина обратная сопротивлению, она измеряется в Сименсах, ранее назывшихся mho. Применительно к сыпучим веществам удобнее говорить об особой проводимости, обычно называемой удельной проводимостью.
Удельная проводимость — это проводимость, измеренная между противоположными сторонами куба вещества со стороной 1 см. Единицей данного типа измерений является Сименс/см. При измерении проводимости воды чаще используются более точные мкС/см (микросименс) и мС/см (миллисименс) .
Соответствующие единицы измерения сопротивления (или удельного сопротивления) — Ом/см, МегаОм/см и килоОм/см. При измерении сверхчистой воды чаще используют МегаОм/см, так как это дает более точные результаты. Сопротивление менее чистой воды, как например, водопроводной, измеряют в килоОм/см.

4) Общее сопротивление при последовательном соединении равно сумме сопротивлений Rсумм=R1+R2+R3…
Ток через все сопротивления протекает один (I). Поэтому ток вычисляешь как Отношение напряжения источника U к Rсумм.

Мощность

P=U*I или P=I*I*R (так как U=I*R).

P1=I*I*R1
P2=I*I*R2
P3=I*I*R3

5) мощность электрического тока в цепи, состоящей из параллельно соединенных участков,
равна сумме мощностей на отдельных участках:

При параллельном соединении каждая лампа подсоединяется на своё номинальное напряжение 220 В. при этом в каждой лампе появляется свой номинальный ток, обеспечивающий заданное свечение в соответствии с номинальной мощностью. мощность зависит от сопротивления нити накаливания. чем больше сопротивление нити, тем меньше ток и соответственно меньше номинальная мощность.
при последовательном соединении ток идёт один и тот же в каждой лампе. а напряжение распределяется в зависимости от доли сопротивления каждой лампы по отношению к сопротивлению всей цепи.
для цепи из двух ламп общее напряжение делится.
напряжение на лампе 40 Вт будет 220Х60:(40+60)=132; В.
напряжение на лампе 60 Вт будет 220Х40:(40+60)=80; В.

Это очень серьезная тема. Давайте рассмотрим ее внимательно. По сути, все просто. Общее сопротивление электрической цепи – это сопротивление, которое цепь оказывает напряжению. Но на этом простота заканчивается. Есть два типа напряжения: переменное и постоянное. Есть два типа сопротивления: активное и реактивное (которое в свою очередь делится на ёмкостное и индуктивное). Активное сопротивление не зависит от изменения частоты сети, ему все равно, какой ток будет протекать: постоянный или переменный. А вот реактивное сопротивление зависит от частоты, причем ёмкостное () или индуктивное ( , дроссели, электродвигатели и т.д.) очень сильно зависят и ведут себя по-разному. Так же, в общее сопротивление входит сопротивление проводов. Сопротивления могут включаться как параллельно, так и последовательно, что тоже имеет свои последствия.

Так чему же равно общее сопротивление электрической цепи?

Для начала окунемся в основы электротехники с головой.

Индуктивное сопротивление. XL= ω·L, где L –индуктивность, ω=2π·f, где f – частота сети. Как видим, с увеличением частоты растет и сопротивление, таким образом, для напряжения высокой частоты катушка становится очень большим сопротивлением.

Емкостное сопротивление Xc= 1/ω·C, где C – емкость. А из этой формулы видно, что сопротивление конденсатора уменьшается, если частота сети повышается. Таким образом, для постоянного тока конденсатор становится бесконечно большим сопротивлением.

Как я и обещал в начале статьи, индуктивное и емкостное сопротивление при переменном токе ведут себя по-разному.

Общее сопротивление индуктивности состоит из двух сопротивлений активного (это сопротивление обмотки постоянному напряжению) и индуктивного, которое мы уже рассмотрели выше. 2, где Z – общее сопротивление цепи, XA – сопротивление катушки постоянному току (проще выражаясь, сопротивление обмотки, которое мы измеряем тестером), XL – индуктивное сопротивление.

Теперь, когда мы разобрались с такими сложными сопротивлениями, как емкостное и индуктивное и можем их привести ко вполне определенному виду можно приступать к другой, не менее интересной части.

Последовательное и параллельное соединение.

В последовательном соединении все довольно просто.

Берем один резистор, к нему второй, третий и т.д. и значения всех резисторов просто складываем друг с другом. Получаем общее сопротивление. То есть, если мы включим 10 резисторов по 10 Ом последовательно, то получим сопротивление 100 Ом. Rобщ=R1+R2+…+Rn

В параллельном все чуть сложнее. Чтобы было понятно, представьте себе длинный широкий коридор, который поделен на десять отсеков. Чтобы пройти из одной части коридора в другую, надо пройти через дверь. Чтобы дойти до конца, надо пройти через все двери – это последовательное соединение. А в параллельном у вас есть точка А и точка Б, и вы можете пройти через 10 дверей из одной точки в другую. Если одна дверь шире, а другая уже, то через широкую дверь пройдет больше людей, чем через узкую. Такой же принцип и у параллельного соединения сопротивлений. Если сопротивления одинаковые по значению, нужно сопротивление одного резистора разделить на их количество. Если мы соединим параллельно 10 резисторов по 10 Ом, то получим сопротивление 1 Ом. Rобщ=(R1+R2+…+Rn)/n, где n – количество резисторов. Если же резисторы не имеют одинакового значения, то формула будет выглядеть так:

Если использовать только два сопротивления, то есть упрощенная формула:

Таким образом, чем больше сопротивлений включено параллельно, тем меньше будет общее сопротивление цепи.

Подведем итог. Для подсчета общего сопротивления цепи надо привести ёмкостные и индуктивные сопротивления к эквивалентному сопротивлению, нарисовать схему. И начнем считать.

Как определить общее сопротивление в параллельной цепи.

Чем отличается последовательное соединение от параллельного? Параллельное соединение сопротивлений


Параллельное и последовательное соединение проводников – способы коммутации электрической цепи. Электрические схемы любой сложности можно представить посредством указанных абстракций.

Определения

Существует два способа соединения проводников, становится возможным упростить расчет цепи произвольной сложности:

  • Конец предыдущего проводника соединен непосредственно с началом следующего — подключение называют последовательным. Образуется цепочка. Чтобы включить очередное звено, нужно электрическую схему разорвать, вставив туда новый проводник.
  • Начала проводников соединены одной точкой, концы – другой, подключение называется параллельным. Связку принято называть разветвлением. Каждый отдельный проводник образует ветвь. Общие точки именуются узлами электрической сети.

На практике чаще встречается смешанное включение проводников, часть соединена последовательно, часть – параллельно. Нужно разбить цепь простыми сегментами, решать задачу для каждого отдельно. Сколь угодно сложную электрическую схему можно описать параллельным, последовательным соединением проводников. Так делается на практике.

Использование параллельного и последовательного соединения проводников

Термины, применяемые к электрическим цепям

Теория выступает базисом формирования прочных знаний, немногие знают, чем напряжение (разность потенциалов) отличается от падения напряжения. В терминах физики внутренней цепью называют источник тока, находящееся вне – именуется внешней. Разграничение помогает правильно описать распределение поля. Ток совершает работу. В простейшем случае генерация тепла согласно закону Джоуля-Ленца. Заряженные частицы, передвигаясь в сторону меньшего потенциала, сталкиваются с кристаллической решеткой, отдают энергию. Происходит нагрев сопротивлений.

Для обеспечения движения нужно на концах проводника поддерживать разность потенциалов. Это называется напряжением участка цепи. Если просто поместить проводник в поле вдоль силовых линий, ток потечет, будет очень кратковременным. Процесс завершится наступлением равновесия. Внешнее поле будет уравновешено собственным полем зарядов, противоположным направлением. Ток прекратится. Чтобы процесс стал непрерывным, нужна внешняя сила.

Таким приводом движения электрической цепи выступает источник тока. Чтобы поддерживать потенциал, внутри совершается работа. Химическая реакция, как в гальваническом элементе, механические силы – генератор ГЭС. Заряды внутри источника движутся в противоположную полю сторону. Над этим совершается работа сторонних сил. Можно перефразировать приведенные выше формулировки, сказать:

  • Внешняя часть цепи, где заряды движутся, увлекаемые полем.
  • Внутренняя часть цепи, где заряды движутся против напряженности.

Генератор (источник тока) снабжен двумя полюсами. Обладающий меньшим потенциалом называется отрицательным, другой – положительным. В случае переменного тока полюсы непрерывно меняются местами. Непостоянно направление движения зарядов. Ток течет от положительного полюса к отрицательному. Движение положительных зарядов идет в направлении убывания потенциала. Согласно этому факту вводится понятие падения потенциала:

Падением потенциала участка цепи называется убыль потенциала в пределах отрезка. Формально это напряжение. Для ветвей параллельной цепи одинаково.

Под падением напряжения понимается и нечто иное. Величина, характеризующая тепловые потери, численно равна произведению тока на активное сопротивление участка. Законы Ома, Кирхгофа, рассмотренные ниже, формулируются для этого случая. В электрических двигателях, трансформаторах разница потенциалов может значительно отличаться от падения напряжения. Последнее характеризует потери на активном сопротивлении, тогда как первое учитывает полную работу источника тока.

При решение физических задач для упрощения двигатель может включать в свой состав ЭДС, направление действия которой противоположно эффекту источника питания. Учитывается факт потери энергии через реактивную часть импеданса. Школьный и вузовский курс физики отличается оторванностью от реальности. Вот почему студенты, раскрыв рот, слушают о явлениях, имеющих место в электротехнике. В период, предшествующий эпохе промышленной революции, открывались главные законы, ученый должен объединять роль теоретика и талантливого экспериментатора. Об этом открыто говорят предисловия к трудам Кирхгофа (работы Георга Ома на русский язык не переведены). Преподаватели буквально завлекали люд дополнительными лекциями, сдобренными наглядными, удивительными экспериментами.

Законы Ома и Кирхгофа применительно к последовательному и параллельному соединению проводников

Для решения реальных задач используются законы Ома и Кирхгофа. Первый выводил равенство чисто эмпирическим путем – экспериментально – второй начал математическим анализом задачи, потом проверил догадки практикой. Приведем некоторые сведения, помогающие решению задачи:

Посчитать сопротивления элементов при последовательном и параллельном соединении

Алгоритм расчета реальных цепей прост. Приведем некоторые тезисы касательно рассматриваемой тематики:

  1. При последовательном включении суммируются сопротивления, при параллельном — проводимости:
    1. Для резисторов закон переписывается в неизменной форме. При параллельном соединении итоговое сопротивление равняется произведению исходных, деленному на общую сумму. При последовательном – номиналы суммируются.
    2. Индуктивность выступает реактивным сопротивлением (j*ω*L), ведет себя, как обычный резистор. В плане написания формулы ничем не отличается. Нюанс, для всякого чисто мнимого импеданса, что нужно умножить результат на оператор j, круговую частоту ω (2*Пи*f). При последовательном соединении катушек индуктивности номиналы суммируются, при параллельном – складываются обратные величины.
    3. Мнимое сопротивление емкости записывается в виде: -j/ω*С. Легко заметить: складывая величины последовательного соединения, получим формулу, в точности как для резисторов и индуктивностей было при параллельном. Для конденсаторов все наоборот. При параллельном включении номиналы складываются, при последовательном – суммируются обратные величины.

Тезисы легко распространяются на произвольные случаи. Падение напряжения на двух открытых кремниевых диодах равно сумме. На практике составляет 1 вольт, точное значение зависит от типа полупроводникового элемента, характеристик. Аналогичным образом рассматривают источники питания: при последовательном включении номиналы складываются. Параллельное часто встречается на подстанциях, где трансформаторы ставят рядком. Напряжение будет одно (контролируются аппаратурой), делятся между ветвями. Коэффициент трансформации строго равен, блокируя возникновение негативных эффектов.

У некоторых вызывает затруднение случай: две батарейки разного номинала включены параллельно. Случай описывается вторым законом Кирхгофа, никакой сложности представить физику не может. При неравенстве номиналов двух источников берется среднее арифметическое, если пренебречь внутренним сопротивлением обоих. В противном случае решаются уравнения Кирхгофа для всех контуров. Неизвестными будут токи (всего три), общее количество которых равно числу уравнений. Для полного понимания привели рисунок.

Пример решения уравнений Кирхгофа

Посмотрим изображение: по условию задачи, источник Е1 сильнее, нежели Е2. Направление токов в контуре берем из здравых соображений. Но если бы проставили неправильно, после решения задачи один получился бы с отрицательным знаком. Следовало тогда изменить направление. Очевидно, во внешней цепи ток течет, как показано на рисунке. Составляем уравнения Кирхгофа для трех контуров, вот что следует:

  1. Работа первого (сильного) источника тратится на создание тока во внешней цепи, преодоление слабости соседа (ток I2).
  2. Второй источник не совершает полезной работы в нагрузке, борется с первым. Иначе не скажешь.

Включение батареек разного номинала параллельно является безусловно вредным. Что наблюдается на подстанции при использовании трансформаторов с разным передаточным коэффициентом. Уравнительные токи не выполняют никакой полезной работы. Включенные параллельно разные батарейки начнут эффективно функционировать, когда сильная просядет до уровня слабой.

Параллельное соединение электрических элементов (проводников, сопротивлений, емкостей, индуктивностей) — это такое соединение, при котором подключенные элементы цепи имеют два общих узла подключения.

Другое определение: сопротивления подключены параллельно, если они подключены одно и той же паре узлов.

Графическое обозначение схемы параллельного соеднинения

На приведенном рисунке показана схема параллельное подключения сопротивлений R1, R2, R3, R4. Из схемы видно, что все эти четыре сопротивления имеют две общие точки (узла подключения).

В электротехнике принято, но не строго требуется, рисовать провода горизонтально и вертикально. Поэтому эту же схему можно изобразить, как на рисунке ниже. Это тоже параллельное соединение тех же самых сопротивлений.

Формула для расчета параллельного соединения сопротивлений

При параллельном соединении обратная величина от эквивалентного сопротивления равна сумме обратных величин всех параллельно подключенных сопротивлений. Эквивалентная проводимость равна сумме всех параллельно подключенных проводимостей электрической схемы.

Для приведенной выше схемы эквивалентное сопротивление можно рассчитать по формуле:

В частном случае при подключении параллельно двух сопротивлений:

Эквивалентное сопротивление цепи определяется по формуле:

В случае подключения «n» одинаковых сопротивлений, эквивалентное сопротивление можно рассчитать по частной формуле:

Формулы для частного рассчета вытекают из основной формулы.

Формула для расчета параллельного соединения емкостей (конденсаторов)

При параллельном подключении емкостей (конденсаторов) эквивалентная емкость равна сумме параллельно подключенных емкостей:

Формула для расчета параллельного соединения индуктивностей

При параллельном подключении индуктивностей, эквивалентная индуктивность рассчитывается так же, как и эквивалентное сопротивление при параллельном соединении:

Необходимо обратить внимание, что в формуле не учтены взаимные индуктивности.

Пример свертывания параллельного сопротивления

Для участка электрической цепи необходимо найти параллельное соединение сопротивлений выполнить их преобразование до одного.

Из схемы видно, что параллельно подключены только R2 и R4. R3 не параллельно, т.к. одним концом оно подключено к E1. R1 — одним концом подключено к R5, а не к узлу. R5 — одним концом подключено к R1, а не к узлу. Можно так же говорить, что последовательное соединение сопротивлений R1 и R5 подключено параллельно с R2 и R4.

Ток при параллельном соединении

При параллельном соединении сопротивлений ток через каждое сопротивление в общем случае разный. Величина тока обратно пропорциональна величине сопротивления.

Напряжение при параллельном соединении

При параллельном соединении разность потенциалов между узлами, объединяющими элементы цепи, одинакова для всех элементов.

Применение параллельного соединения

1. В промышленности изготавливаются сопротивления определенных величин. Иногда необходимо получить значение сопротивления вне данных рядов. Для этого можно подключить несколько сопротивлений параллельно. Эквивалентное сопротивление всегда будет меньше самого большого номинала сопротивления.

2. Делитель токов.

Ни одна операция в электронике или электротехнике не обходится без вычисления сопротивления. В этом случае рассматривают только тот участок цепи, в котором находится смешанное соединение резисторов. Инженерам и физикам необходимо понимать то, как именно происходят расчёты в таких схемах. Всего разделяют несколько видов подключения, которые используются в цепях различной сложности.


Последовательное соединение

Выделяют такие способы соединения резисторов
: последовательное, параллельное и комбинированное. При последовательном подключении конец первого резистора подключают к началу второго, его часть к третьему. Так действуют со всеми составляющими. То есть все компоненты цепи следуют друг за другом. Через них в таком подключении будет проходить один общий электрический ток. Для таких схем физики применяют формулу, в которой между точками А и В есть только один путь протекания заряженных электронов.

От количества подключённых резисторов зависит сопротивление протекающему электричеству. Чем больше составляющих, тем оно выше. Его рассчитывают по формуле
: R общее = R1+R2+…+Rn, где:

  • R общее — это сумма всех сопротивлений;
  • R1 — первый резистор;
  • R2 — второй компонент;
  • Rn — последняя составляющая в цепи.

Параллельное подключение

Параллельное соединение подразумевает подключение начал резисторов к одной точке
, а концов к другой. Сами компоненты при этом расположены на одинаковом расстоянии друг от друга, а их количество не ограничено. По каждой составляющей электричество протекает отдельно, выбирая один из нескольких путей.

Из-за того, что в цепи находится несколько компонентов и путей прохода тока, сопротивление значительно меньше, чем при последовательном соединении. То есть общая сумма противодействия уменьшается пропорционально увеличению количества составляющих. Формула для определения общей суммы противостояния электричеству: 1/R общее = 1/R1+1/R2+…+1/Rn.

В расчётах общее сопротивление всегда должно быть меньше любого из составляющих цепи. Способ вычисления суммы противостояния для схемы из двух резисторов немного отличается: 1/R общее = (R1 х R2)/(R1+R2). Если в системе у компонентов одинаковые показатели сопротивления, то общее число будет равно половине одного из составляющих.

Смешанный вариант

В смешанном соединении сопротивлений комбинируют последовательную и параллельную схему подключений. В этом случае несколько компонентов соединяют одним способом, а другие — вторым, но все они включены в одну цепь. В физике такой метод соединения называют последовательно-параллельным.

Для вычисления суммы противостояния электричеству схему нужно разбить на мелкие участки, в которых резисторы подключены одинаковым способом. Затем расчёты проводят по алгоритму:

  • в цепи с параллельно соединёнными компонентами высчитывают эквивалентное сопротивление;
  • после этого высчитывают противостояние на последовательно подключённых участках схемы;
  • наглядную иллюстрацию нужно перерисовать, обычно получается цепь с последовательным соединением резисторов;
  • рассчитывают сопротивление в новой схеме по одной из двух формул.

Лучше понять методы вычислений поможет пример. Если в схеме всего пять компонентов, они могут располагаться по-разному. Начало первого резистора подключено к точке А, конец — к В. От неё идёт отдельная схема с комбинированным соединением. Вторая и третья составляющие находятся на последовательной линии, четвёртый компонент параллелен им. От конечной точки этой цепи — Г — исходит последний резистор.

Сначала высчитывают сумму сопротивления последовательного участка внутренней схемы
: R2+R3. После этого цепь перерисовывают так, чтобы второй и третий компоненты были соединены в один. В результате внутренняя цепь имеет параллельное подключение. Теперь высчитывают её противостояние: (R2,3xR4)/(R2,3+R4). Можно второй раз изобразить полученную цепь.

В схеме будет три резистора, соединённые последовательным методов. Причём средний включает параметры второго, третьего и четвёртого компонента.

Теперь можно узнать общую сумму сопротивлений. Для этого складывают показатели противостояний электричеству первого, пятого и остальных составляющих. Формула будет иметь вид: R1+(R2,3xR4)/(R2,3+R4)+R5. Можно сразу подставить в неё все параметры компонентов.

На практике последовательный и параллельный метод соединения используются редко, ведь в приборах схемы обычно сложные. Поэтому в цепях резисторы часто соединены комбинированным способом. Сопротивление в таких случаях высчитывают пошагово.

Если сразу выводить числа в общую формулу, то можно ошибиться и получить неверные результаты. А это может отрицательно сказаться на работе электрического прибора.

Последовательное и параллельное соединение резисторов

Последовательное соединение резисторов

Последовательное соединение

это соединение двух или более резисторов в форме цепи, в которой каждый отдельный резистор соединяется с другим отдельным резистором только в одной точке.

Общее сопротивление Rобщ

При таком соединении, через все резисторы проходит один и тот же электрический ток. Чем больше элементов на данном участке электрической цепи, тем «труднее» току протекать через него. Следовательно, при последовательном соединении резисторов их общее сопротивление увеличивается, и оно равно сумме всех сопротивлений.

Напряжение при последовательном соединении

Напряжение при последовательном соединении распределяется на каждый резистор согласно закону Ома:

Т.е чем большее сопротивление резистора, тем большее напряжение на него падает.

Параллельное соединение резисторов

Параллельное соединение

это соединение, при котором резисторы соединяются между собой обоими контактами. В результате к одной точке (электрическому узлу) может быть присоединено несколько резисторов.

Общее сопротивление Rобщ

При таком соединении, через каждый резистор потечет отдельный ток. Сила данного тока будет обратно пропорциональна сопротивлению резистора. В результате общая проводимость такого участка электрической цепи увеличивается, а общее сопротивление в свою очередь уменьшается.

Таким образом, при параллельном подсоединении резисторов с разным сопротивлением, общее сопротивление будет всегда меньше значения самого маленького отдельного резистора.

Формула общей проводимости при параллельном соединении резисторов:

Формула эквивалентного общего сопротивления при параллельном соединении резисторов:

Для двух одинаковых резисторов общее сопротивление будет равно половине одного отдельного резистора:

Соответственно, для n одинаковых резисторов общее сопротивление будет равно значению одного резистора, разделенного на n.

Напряжение при параллельном соединении

Напряжение между точками A и B является как общим напряжением для всего участка цепи, так и напряжением, падающим на каждый резистор в отдельности. Поэтому при параллельном соединении на все резисторы упадет одинаковое напряжение.

Электрический ток при параллельном соединении

Через каждый резистор течет ток, сила которого обратно пропорциональна сопротивлению резистора. Для того чтобы узнать какой ток течет через определенный резистор, можно воспользоваться законом Ома:

Смешанное соединение резисторов

Смешанным соединением называют участок цепи, где часть резисторов соединяются между собой последовательно, а часть параллельно. В свою очередь, смешанное соединение бывает последовательного и параллельного типов.

Общее сопротивление Rобщ

o
Цепь разбивают на участки с только пареллельным или только последовательным соединением. o
Вычисляют общее сопротивление для каждого отдельного участка.

o
Вычисляют общее сопротивление для всей цепи смешанного соединения.

Так это будет выглядеть для схемы 1:

Также существует более быстрый способ расчета общего сопротивления для смешанного соединения. Можно, в соответствии схеме, сразу записывать формулу следующим образом:

o
Если резисторы соединяются последоватеьно — складывать.

o
Если резисторы соединяются параллельно — использовать условное обозначение «||». o
Подставлять формулу для параллельного соединения где стоит символ «||».

Так это будет выглядеть для схемы 1.

При решении задач принято преобразовывать схему, так, чтобы она была как можно проще. Для этого применяют эквивалентные преобразования. Эквивалентными называют такие преобразования части схемы электрической цепи, при которых токи и напряжения в не преобразованной её части остаются неизменными.

Существует четыре основных вида соединения проводников: последовательное, параллельное, смешанное и мостовое.

Последовательное соединение

Последовательное соединение
– это такое соединение, при котором сила тока на всем участке цепи одинакова. Ярким примером последовательного соединения является старая елочная гирлянда. Там лампочки подключены последовательно, друг за другом. Теперь представьте, одна лампочка перегорает, цепь нарушена и остальные лампочки гаснут. Выход из строя одного элемента, ведет за собой отключение всех остальных, это является существенным недостатком последовательного соединения.

При последовательном соединении сопротивления элементов суммируются.

Параллельное соединение

Параллельное соединение
– это соединение, при котором напряжение на концах участка цепи одинаково. Параллельное соединение наиболее распространено, в основном потому, что все элементы находятся под одним напряжением, сила тока распределена по-разному и при выходе одного из элементов все остальные продолжают свою работу.

При параллельном соединении эквивалентное сопротивление находится как:

В случае двух параллельно соединенных резисторов

В случае трех параллельно подключенных резисторов:

Смешанное соединение

Смешанное соединение
– соединение, которое является совокупностью последовательных и параллельных соединений. Для нахождения эквивалентного сопротивления нужно, “свернуть” схему поочередным преобразованием параллельных и последовательных участков цепи.

Сначала найдем эквивалентное сопротивление для параллельного участка цепи, а затем прибавим к нему оставшееся сопротивление R 3 . Следует понимать, что после преобразования эквивалентное сопротивление R 1 R 2 и резистор R 3 , соединены последовательно.

Итак, остается самое интересное и самое сложное соединение проводников.

Мостовая схема

Мостовая схема соединения представлена на рисунке ниже.

Для того чтобы свернуть мостовую схему, один из треугольников моста, заменяют эквивалентной звездой.

И находят сопротивления R 1 , R 2 и R 3 .

Контрольная работа №2 по теме «Работа и мощность тока» Вариант 1

Контрольная работа №2 по теме

«Работа и мощность тока» Вариант 1

1. Определите общее сопротивление цепи и показания амперметра, если R1 = 5 Ом, R2 = 10 Ом, R3 = 15 Ом, а показание вольтметра 60 В.

R1 R2 R3

2. Какое количество теплоты выделит за 5 мин проволочная спираль сопротивлением 30 Ом, если сила тока в ней равна 2 А?

3. Электропаяльник мощностью120 Вт рассчитан на напряжение 220 В. Определите силу тока в обмотке паяльника и ее сопротивление.

4. Определите общее сопротивление цепи,

если R1 = 4 Ом, R2 = 6 Ом, R3 = 15 Ом, R4 = 5 Ом

R1 R2

R4

R3

Контрольная работа №2 по теме

«Работа и мощность тока» Вариант 2

1. Определите общее сопротивление цепи и показания амперметра, если R1 = 8 Ом,

R2 = 8 Ом, а показание вольтметра 80 В.

R1

R2

2. Определите мощность тока в электрической лампочке, если при напряжении 3 В, сила тока в ней равна 0,1 А?

3. Какое количество теплоты выделит за 10 с проводник сопротивлением 50 Ом при напряжении в сети 50 В ?

4. Определите общее сопротивление цепи,

если R1 = 4 Ом, R2 = 3 Ом, R3 = 6 Ом, R4 = 5 Ом

R2

R1 R4

R3

Контрольная работа №2 по теме

«Работа и мощность тока» Вариант 1

1. Определите общее сопротивление цепи и показания амперметра, если R1 = 5 Ом, R2 = 10 Ом, R3 = 15 Ом, а показание вольтметра 60 В.

R1 R2 R3

2. Какое количество теплоты выделит за 5 мин проволочная спираль сопротивлением 30 Ом, если сила тока в ней равна 2 А?

3. Электропаяльник мощностью120 Вт рассчитан на напряжение 220 В. Определите силу тока в обмотке паяльника и ее сопротивление.

4. Определите общее сопротивление цепи,

если R1 = 4 Ом, R2 = 6 Ом, R3 = 15 Ом, R4 = 5 Ом

R1 R2

R4

R3

Контрольная работа №2 по теме

«Работа и мощность тока» Вариант 2

1. Определите общее сопротивление цепи и показания амперметра, если R1 = 8 Ом,

R2 = 8 Ом, а показание вольтметра 80 В.

R1

R2

2. Определите мощность тока в электрической лампочке, если при напряжении 3 В, сила тока в ней равна 0,1 А?

3. Какое количество теплоты выделит за 10 с проводник сопротивлением 50 Ом при напряжении в сети 50 В ?

4. Определите общее сопротивление цепи,

если R1 = 4 Ом, R2 = 3 Ом, R3 = 6 Ом, R4 = 5 Ом

R2

R1 R4

R3

Сопротивление при параллельном соединении, формула для расчета сопротивления при параллельном соединении

В этой статье мы разберем, как посчитать общее сопротивление при параллельном соединении сопротивлений. Параллельным соединением сопротивлений называется соединение (рисунок ниже), при котором один зажим каждого из сопротивлений присоединяется к одной точке (узлу) электрической цепи, а другой зажим каждого из тех же сопротивлений присоединяется к другой точке электрической цепи. Таким образом, между двумя точками (узлами) электрической цепи включается несколько сопротивлений, образующих параллельные ветви.

При этом напряжение между концами всех ветвей будет одним и тем же, а токи в отдельных ветвях определяются по закону Ома:
I1 = U / r1 ; I2 = U / r2 ; I3 = U / r3.

Напряжение U между узлами (А и Б):
U = I1r1 = I2r2 = I3r3,
откуда
I1 / I2 = R2 / R1  и  I2 / I3 = R3 / R2,
т. е.

Токи в параллельных ветвях распределяются обратно пропорционально их сопротивлениям.

Согласно первому правилу Кирхгофа,
I = I1 + I2 + I3
или
U / Rсум = U / R1 + U / R2 + U / R2 = U (1 / R1 + 1 / R2 + 1 / R3).
Произведя сокращение на U, получим:
1 / Rсум = 1 / R1 + 1 / R2 + 1 / R3
или
g = g1 + g2 + g3 ,

где R и g—сопротивление и проводимость разветвленной цепи или, как их часто называют, общие сопротивление и проводимость цепи.
Из полученной формулы следует, что

Общая проводимость разветвленной цепи равна сумме проводимостей отдельных ветвей.

Формула
1 / Rсум = 1 / R1 + 1 / R2 + 1 / R3
дает возможность определить общее сопротивление цепи. Например, для трех параллельно соединенных сопротивлений, приведя правую часть уравнения к общему знаменателю, получим:
1 / Rсум = R2R3 + R1R3 + R1R2 / R1R2R3
откуда
Rсум = R1R2R3 / R2R3 + R1R3 + R1R2
Если сопротивления R1 = R2 = R3, то общее сопротивление цепи:
Rсум = R1 / 3,
а в общем случае при n параллельных ветвях с равными сопротивлениями R1 :
Rсум = R1 / n
В случае двух параллельных ветвей:
1 / Rсум = 1 / R1 + 1 / R2
откуда
Rсум = R1R2 / R2R3 + R1R3

При параллельном соединении приемников энергии все они находятся под одним и тем же напряжением, и режим работы каждого из них не зависит от остальных. Совершенно иначе обстоит дело при последовательном соединении приемников, при котором изменение сопротивления одного из них тотчас же приводит к изменению напряжения на других, последовательно соединенных с ним. Поэтому электрические лампы и двигатели, предназначенные для работы при определенном (номинальном) напряжении, включаются параллельно. Одинаковые электрические лампы иногда соединяются последовательно. Пусть, например, напряжение сети U, а напряжение лампы U0 < U Тогда n ламп соединяются цепочкой друг за другом, причем n > U / U0
Такое соединение ламп можно встретить, например, в трамваях, метро и других случаях.

Пример 1:
К сети с напряжением 220 в параллельно подключены двигатель мощностью 1,1 квт и 11 ламп, каждая мощностью 40 вт. Определить ток в главных (подводящих) проводах

Ток двигателя
I1 = P1 / U = 1100 / 220 = 5a.
Ток ламп
I2 = P2 / U = 11 x 40 / 220 = 2a.
Ток в подводящих проводах
I = I1 + I2 = 5 + 2 = 7a.

Пример 2:
Определить общее сопротивление десяти параллельно включенных ламп накаливания, если каждая из них 240 ом:
R = Rл / n = 240 / 10 = 24ом.

Определите общее сопротивление rab электрической цепи схема

К выполнению контрольной работы

МЕТОДИЧЕСКИЕ УКАЗАНИЯ

Пример методики решения задачи №1. «Расчет цепей постоянного тока»

Для схемы, приведенной на рисунке 1.1, заданы сопротивления резисторов и ток I4 в резисторе R4. Определить:

1.эквивалентное сопротивление цепи RAB,

2. токи в каждом резисторе,

3. напряжение UAB, приложенное к цепи.

Решение. Задача относится к теме «Электрические цепи постоянного тока» После усвоения условия задачи приводим поэтапное решение, предварительно обозначив стрелкой направление тока в каждом резисторе. Индекс тока должен соответствовать номеру резистора, по которому он проходит.

1.1. Определяем общее сопротивление разветвления R2, R3. Резисторы соединены

= Ом

Теперь схема принимает вид, показанный на рисунке 1.2

1.2 Резисторы R2,3 и R5 соединены последовательно, их общее сопротивление

Соответствующая схема приведена на рисунке 1.3

1.2. Резисторы R2,3,5 и R4 соединены параллельно, их общее сопротивление

Ом

Теперь схема цепи имеет вид, приведенный на рисунке 1.4

1.3. Находим эквивалентное сопротивление всей цепи:

2.Определение токов в резисторах

2.1.Зная силу тока I4, находим напряжение на резисторе R4:

Это же напряжение приложено к резисторам R2,3 +R5 (рисунок 1.2.). Поэтому ток в

2.2 Находим падение напряжения на резисторе R5:

Поэтому напряжение на резисторах R2 и R3:

2.3 Определяем токи в резисторах R2 и R3 :

Применяя закономерности параллельного соединения резисторов, находим ток в резисторе R1:

2. 4. Вычисляем падение напряжения на резисторе R1:

3. Находим напряжение UAB, приложенное ко всей цепи:

Задача 1 (Варианты 01 – 30)

Цепь постоянного тока содержит шесть резисторов, соединенных смешанно. Схема цепи и значения резисторов указаны на соответствующем рисунке. Номер рисунка и величина одного из заданных токов или напряжений приведена в таблице 3. Индекс тока или напряжения совпадает с индексом резистора, по которому проходит этот ток или на котором действует указанное напряжение. Например, через резистор R5 проходит ток I5 и на нем действует напряжение U5. Определить:

1) эквивалентное сопротивление цепи относительно вводов АВ;

2) ток в каждом резисторе;

3) напряжение на каждом резисторе;

4) расход электрической энергии цепью за 10 часов.

Электротехника и электроника

Контрольные задания и методические указания

к выполнению контрольной работы № 1

для студентов заочного отделения

«Разработка и эксплуатация нефтяных и газовых месторождений»

ОБЩИЕ МЕТОДИЧЕСКИЕ УКАЗАНИЯ

Материал, изучаемый по учебнику, необходимо конспектировать в тетради. Основные определения следует подчеркивать, формулы обводить. Электрические схемы должны быть вычерчены в условных обозначениях, соответствующим ГОСТам. После проработки какой-либо темы необходимо без помощи учебника вывести доказательства законов и формул. Нельзя ничего оставлять непонятным при изучении предмета; если самому преодолеть затруднение не удается, необходимо обратиться за консультацией к преподавателю. Серьезное внимание должно быть уделено задачам и вопросам для самопроверки, а также разбору решений типовых примеров, помещенных в учебнике и настоящем пособии.

УКАЗАНИЯ К ВЫПОЛНЕНИЮ КОНТРОЛЬНЫХ РАБОТ

Каждая контрольная работа по дисциплине «Электротехника и электроника» содержит четыре задачи. Варианты для каждого учащегося индивидуальные. Номер варианта определяется по порядковому номеру в журнале успеваемости и посещаемости учащегося (для номеров с 1 по 30), если порядковый номер учащегося в журнале больше 30, номер варианта начинается с номера 1 (например, по журналу учащийся имеет номер 31, следовательно, должен выбрать вариант № 1). В табл. 1 учащийся находит номера задач, которые он должен решить по варианту.

Задачи, выполненные не по своему варианту, не засчитываются и возвращаются учащемуся.

Контрольная работа выполняется в отдельной тетради в клетку. Условия задач следует переписывать полностью. Необходимо оставлять поля шириной 25-30 мм для замечаний рецензента, а в конце тетради 2-3 страницы для рецензии. Страницы тетради обязательно должны быть пронумерованы. Формулы и расчеты пишут чернилами, а чертежи и схемы выполняют карандашом, на графиках и векторных диаграммах указывают масштаб. Решение задач обязательно ведут в Международной системе единиц (СИ).

После получения работы с оценкой и замечаниями преподавателя надо исправить отмеченные ошибки, выполнить все его указания и повторить недостаточно усвоенный материал. Если контрольная работа получила неудовлетворительную оценку, то учащийся выполняет её снова и отправляет на повторную проверку. В случае возникновения затруднений при выполнении контрольной работы учащийся должен обратиться в техникум для получения письменной или устной консультации.

№ вариантаНомер задач для контрольных работ № 1
1, 2, 3, 4
1, 2, 3, 5
1, 2, 3, 6
1, 2, 3, 7
1, 2, 3, 8
1, 2, 3, 9
1, 2, 3, 10
1, 2, 3, 11
1, 2, 3, 12
1, 2, 3, 13
1, 2, 3, 4
1, 2, 3, 5
1, 2, 3, 6
1, 2, 3, 7
1, 2, 3, 8
1, 2, 3, 9
1, 2, 3, 10
1, 2, 3, 11
1, 2, 3, 12
1, 2, 3, 13
1, 2, 3, 4
1, 2, 3, 5
1, 2, 3, 6
1, 2, 3, 7
1, 2, 3, 8
1, 2, 3, 9
1, 2, 3, 10
1, 2, 3, 11
1, 2, 3, 12
1, 2, 3, 13

Таблица№1

Методические указания к выполнению контрольной работы

В контрольную работу входят «Введение» и шесть тем. По каждой теме предусмотрены задачи. В таблице №2 указаны номера задач к соответствующей теме и номера таблиц с данными к этим задачам. Схемы и векторные диаграммы должны выполняться с помощью чертежных инструментов.

Таблица №2

№ темыНаименование темы№ задач№ таблиц
Электрические цепи постоянного тока
Однофазные электрические цепи переменного тока2-34-5
Трехфазные цепи4-66-8

Методические указания к решению задачи № 1

Решение этой задачи требует знание законов Ома для всей цепи и его участков, первого и второго законов Кирхгофа и методики определения эквивалентного сопротивления цепи при смешанном соединении резисторов. Содержание задачи и схемы цепей с соответствующими данными приведены в условии и таблице №3. Перед решением задачи рассмотрите типовой пример№1.

Пример 1 Для схемы, приведенной на рис. 1, определить эквивалентное сопротивление цепи RAB, токи в каждом резисторе и напряжение UAB, приложенной к цепи.

Заданы сопротивления резисторов и ток I4 в резисторе R4. Как изменятся токи в резисторах при: а) замыкание рубильника Р1; б) расплавление ставки предохранителя Пр4?

В обоих случаях напряжение UABостается неизменным.

Решение:

Задача относится к теме «Электрические цепи постоянного тока». После усвоения условия задачи проводим поэтапное решение, предварительно обозначив стрелкой направление тока в каждом резисторе. Индекс тока должен соответствовать номеру резистора, по которому он проходит.

1. Определяем общее сопротивление разветвления R2R3. Резисторы соединены параллельно, поэтому

Теперь схема цепи примет вид, показанный на рис.1б

2. Резисторы и R2,R5 соединены последовательно, их общее сопротивление

Соответствующая схема приведена на рис 1в.

3. Резисторы R2,3,5и R4, соединены параллельно, их общее сопротивление

Ом

Теперь схема цепи имеет вид, приведенный на рис. 1, г

4. Находим эквивалентное сопротивление всей цепи:

Ом(рис. 1,д)

5. Зная силу тока I4, находим напряжение на резисторе R4:

В

Это же напряжение приложено к резисторам R2,3иR5(рис.1, б). Поэтому ток в резисторе R5

А

6. Находим падение напряжения на резистореR5

В

Поэтому напряжение на резисторах R2,3,

В

Применяя первый закон Кирхгофа, находим ток в резисторе R1

8. Вычисляем падение напряжения на резисторе R1

9. Находим напряжение Uавприложенное ко всей цепи:

В или В

10. При включении рубильника Р1сопротивление R1 замыкается накоротко и схема цепи имеет вид, показанный на рис. 1, е. Эквивалент­ное сопротивление цепи в этом случае

Ом

Поскольку напряжение Uав остается равным 100 В, можно найти токи в резисторах R4и R5:

;

Определим падение напряжения на резисторе R5

B

Поэтому напряжение на резисторах R2, R3

B

Теперь можно найти токи в резисторах R2и R3:

A;

Проверим правильность вычисление токов, по закону Кирхгофа:

A

A

Таким образом, задача решена верно.

11. При расплавлении предохранителя Пр4 резистор R4выключается и схема принимает вид, показанный на рис. 1, ж.

Вычисляем эквивалентное сопротивление схемы:

Ом

Поскольку напряжение UABостается неизменным, находим токи и I1иI5:

А

В

А; А

Сумма этих токов равна току I1:

А

Задача № 1 (варианты 1-30)

Цепи постоянного тока содержат несколько резисторов, соединенных смешанно. Схема цепи с указанием сопротивлений резисторов приведены на соответствующем рисунке. Номер рисунка, заданные значения одного из них напряжения или тока и величина, подлежащая определению, приведены в таблице №3.

Определить так же мощность, потребляемую всей цепью, и расход электрической энергии цепью за 8 часов.

Пояснить, с помощью логических рассуждений характер измерения электрической величины, заданной в таблице вариантов (увеличится, уменьшится, останется без изменений), если один из резисторов замкнуть накоротко или выключить из схемы. Характер действия с резистором и его действия указаны в таблице №3. При этом считать напряжение UAB неизменным.

Таблица 3

Номер вариантаНомер рисункаЗадаваемая величинаОпреде-литьДействие с резисторомИзменение какой величины рассмотреть
Замыкается накороткоВыключа-ется из схемы
UAB = 100 ВI3R1I5
I1 = 20 АI4R4U5
U2 = 30 ВI6R5I1
I5 = 10 АUABR2I5
UAB = 50 ВI1R2U3
I2 = 3,75 АI5R5U1
I4 = 5 АUABR4I3
U5 = 30 ВI1R3U4
I3 = 1,25 АU1R3I2
UАВ = 80 BU4R4I5
I3 = 1 АU5R2U1
U1 = 20 BI4R5I4
I5 = 5 AUABR5U1
I1 = 12 AI3R2U4
U5 = 60 BI1R1U5
UAB = 5 ВU4R5I3
I2 = 3 АI5R4U1
U2 = 12 BU1R4I5
U4 = 36 BI1R4U5
I4 = 12 АUABR4U5
UAB = 50 BI3R1I6
I2 = 2 AUАВR2U1
I1 = 5 АU4R3I1
U5 = 18 BI1R6I2
I3 = 1,2 АUАВR5U1
I5 = 6 АI1R3U2
UAB = 80 ВI6R1U5
I6 = 3 АU1R5U1
U4 = 10 ВUАВR3I6
U1 = 20 ВI4R2I5

Не нашли то, что искали? Воспользуйтесь поиском:

Лучшие изречения: При сдаче лабораторной работы, студент делает вид, что все знает; преподаватель делает вид, что верит ему. 9396 – | 7310 – или читать все.

91.146.8.87 © studopedia.ru Не является автором материалов, которые размещены. Но предоставляет возможность бесплатного использования. Есть нарушение авторского права? Напишите нам | Обратная связь.

Отключите adBlock!
и обновите страницу (F5)

очень нужно

Расчёт электрических схем, содержащих несколько сопротивлений (резисторов), при нахождении силы тока в цепи, напряжения или мощности, производится с использованием метода свёртывания. Метод заключается в том, чтобы найти эквивалентное сопротивление выделенных участков цепи. Основная задача – замена резисторов, имеющих различное подключение относительно друг друга, на эквивалент (Rэкв.).

Определение эквивалентного сопротивления

При рассмотрении схем любых электрических или электронных устройств можно увидеть, что такие компоненты, как резисторы, имеют разные типы соединений между собой. Чтобы определить эквивалентное соединение, необходимо рассматривать два элемента, включенных в определённом порядке. Несмотря на то, что на чертеже их может быть несколько десятков, и соединены они по-разному, есть только два типа включения их друг с другом: последовательное и параллельное. Остальные конфигурации – это лишь их вариации.

Последовательное соединение элементов

Подобное включение подразумевает комбинацию деталей в прямой последовательности. Выход одного сопротивления подключается к входу другого. При этом отсутствуют какие-либо ответвления на участке. Величина тока, который проходит через все соединённые последовательно компоненты, будет одна и та же.

Внимание! Снижение потенциала на каждом резистивном элементе в сумме даст полное напряжение, приложенное к последовательной цепи.

В случае постоянного тока формула закона Ома для отрезка цепи имеет вид:

Сила тока зависит от приложенного напряжения и оказанного ему сопротивления. Если выразить R, его формула:

Параметры последовательной цепи, включающей n соединённых друг с другом элементов, имеют свои особенности.

Проходящий по цепи ток везде одинаковый:

Прикладываемое напряжение является суммой напряжений на каждом резисторе:

Следовательно, рассчитать можно общее:

Rэкв.= U1/I + U2/I + … +Un/I) = R1 + R2 + … +Rn.

Важно! Последовательная цепь, имеющая в своём составе N резисторов равного номинала, имеет эквивалентное сопротивление Rэкв. = N*R.

Параллельное соединение

Когда условные выходы деталей имеют общий контакт в одной точке (узле) схемы, а условные входы так же объединены во второй, говорят о параллельном соединении. Узел на чертеже обозначается графической точкой. Это место, где происходят разветвления цепей в схемах. Такой вариант подключения резисторов обеспечивает одинаковое падение напряжения U для всех параллельных элементов. Ток в этой позиции будет равен сумме токов, идущих по каждому компоненту.

Когда в параллельное подключение входит n резистивных элементов, то разность потенциалов, ток и общее сопротивление будут иметь следующие выражения:

  • общий ток: I = I1 + I2 + … + In;
  • общее напряжение: U = U1 = U2 = … = Un;
  • Rобщ. = Rэкв. = U/I1 + U/I2 + …+ U/In) = 1/R1 + 1/R2 +…+ 1/Rn.

Величину, обратно пропорциональную сопротивлению 1/R, называют проводимостью.

Если n равных по номиналу сопротивлений включить параллельно, то Rэкв. = (R*R)/n*R = R/n. Формула подходит и для индуктивных сопротивлений проволочных катушек и ёмкостных сопротивлений конденсаторов.

Расчёт при смешанном соединении устройств

Произвести расчет сопротивления цепи, когда она разветвлена и наполнена разными видами резистивных соединений, просто не получится. Затрудняет решение задачи множество участков, где детали подключены друг другу в разных комбинациях. В таких обстоятельствах желательно выполнять ряд преобразований, добиваясь упрощения схемы вводом отдельных эквивалентных элементов. Выявляют при этом подходящие контуры последовательных и параллельных присоединений.

Например, выискав некоторое количество последовательных подключений резисторов, заменяют их на один эквивалентный компонент. Определив элементы, соединённые последовательно, также рисуют вместо него эквивалент. Вновь начинают искать подобные простые соединения.

Метод называют «методом свёртывания». Схему упрощают до тех пор, пока в ней не останется одно Rэкв.

Важно! Метод эквивалентных преобразований применяется тогда, когда питание рассматриваемого участка цепи осуществляется от одного источника электрического тока, а также при определении Rэкв. в замкнутом контуре с одной ЭДС.

Такой относительный способ определения Rэкв используют и для изучения зависимости токов в некоторой цепи от значения R нагрузки. Это метод эквивалентного генератора, при котором сложный двухполюсник, являющийся активным, представляют эквивалентным генератором. При этом считают, что ЭДС его соответствует Uх.х. (холостого хода) на зажимах, R внутреннее соответствует R входному двухполюсника пассивного на тех же зажимах. Для такого определения источники тока разъединяют, а канал ЭДС закорачивают.

Физические формулы и примеры вычислений

Формулы для эквивалентных сопротивлений цепи, состоящей из пары резисторов R1 и R2, можно выделить в определённый ряд:

  • параллельное присоединение определяют по формуле Rэкв. = (R1*R2)/R1+R2;
  • последовательное включение вычисляют, определяя его сумму Rэкв. = R1+R2.

У смешанного соединения резистивных элементов нет конкретной формулы. Чтобы не запутаться при длительных преобразованиях, здесь допустимо воспользоваться специальной программой из интернета. Это сервис «онлайн-калькулятор». Он поможет разобраться со сложными схемами соединения, будь то треугольник, квадрат, пятиугольник или иная схематичная фигура, образованная резистивными элементами.

Понять, как работают все формулы и методы, можно на конкретной задаче. На представленном первом рисунке – смешанная электрическая схема. Она включает в себя 10 резисторов. Элементы представлены в следующих номиналах:

  • R1 = 1 Ом;
  • R2 = 2 Ом;
  • R3 = 3 Ом;
  • R4 = 6 Ом;
  • R5 = 9 Ом;
  • R6 = 18 Ом;
  • R7 = 2Ом;
  • R8 = 2Ом;
  • R9 = 8 Ом;
  • R10 = 4 Ом.

Напряжение, поданное на схему:

Требуется рассчитать токи на всех резистивных элементах.

Для расчётов применяется закон Ома:

I = U/R, подставляя вместо R эквивалентное сопротивление.

Внимание! Для решения этой задачи сначала вычисляют общее (эквивалентное) R, после чего уже рассчитывают ток в цепи и напряжение на каждом резистивном компоненте.

Вычисляя Rэкв., разделяют заданную цепь на звенья, вмещающие в себя параллельные и последовательные включения. Делают расчёты для каждого такого звена, после – всей цепи целиком.

На рисунке выше изображено смешанное соединение сопротивлений. Его можно разбить на три участка:

  • АВ – участок, имеющий две параллельных ветви;
  • ВС – отрезок, вмещающий в себя последовательное сопряжение;
  • CD – отрезок схемы с расположением трёх параллельных цепочек.

Сопротивления R2 и R3, образующие нижнюю ветку отрезка АВ, соединены последовательно, что учитывается при расчёте.

Если посмотреть на участок СD, то можно отметить смешанное включение резистивных элементов.

Начало расчётов состоит в определении эквивалентных сопротивлений для этих смешанных фрагментов. Выполняют это в следующем порядке:

  • Rэкв.2,3 = R2+R3=2 + 3 = 5 Ом;
  • Rэкв.7,8 = (R7*R8)/R7 + R8 = (2*2)/2 + 2 = 1 Ом;
  • Rэкв.7,8,9 = Rэкв.7,8 + R9 = 1 + 8 = 9 Ом.

Зная значения полученных эквивалентов, упрощают первоначальную схему. Она будет иметь вид, представленный на рисунке ниже.

Далее можно уже определить Rэкв. для участков AB, BC, CD, по формулам:

  • Rэкв.AB = (R1*Rэкв 2,3)/R1 + Rэкв 2,3 = (1*5)/1 + 5 = 0,83 Ом;
  • Rэкв.BC = R4 + R5 = 6 + 9 = 15 Ом;
  • 1/Rэкв. CD = 1/R6 + 1/Rэкв.7,8,9 + 1/R10 = 1/18 + 1/9 + 1/4 = 0,05 + 0,11 + 0,25 = 0,41 Ом.

В результате выполненных вычислений получается эквивалентная схема, в которую входят три Rэкв. сопротивления. Она имеет вид, показанный на рисунке ниже.

Теперь можно определить эквивалентное сопротивление всей первоначальной схемы, сложив эквивалентные значения всех трёх участков:

Rэкв. = Rэкв.AB + Rэкв.BC + Rэкв.CD = 0,83 + 15 + 0,41 = 56,83 Ом.

Далее, используя закон Ома, находят ток в последнем последовательном участке:

I = U/ Rэкв. = 24/56,83 = 0,42 А.

Зная силу тока, можно найти, какое падение напряжения на рассмотренных участках AB, BC, CD. Это выполняется следующим образом:

  • UAB = I* Rэкв.AB= 0,42*0,83 = 0,35 В;
  • UBC = I* Rэкв.BC= 0,42*15 = 6,3В;
  • UCD = I* Rэкв.CD = 0,42*0,41 = 0,17 В.

Следующим шагом станет определение токов на параллельных отрезках AB и CD:

  • I1 = UAB/R1 = 0,35/1 = 0,35 А;
  • I2 = UAB/Rэкв. 2,3 = 0,35/5 = 0,07 А;
  • I3 = UCD/R6 = 0,17/18 = 0,009 А;
  • I6 = UCD/Rэкв.7,8,9= 0,17/9 = 0,02 А;
  • I7 = UCD/R10 = 0,17/4 = 0,04 А.

Далее, чтобы найти значения токов, проходящих через R7 и R8, нужно рассчитать напряжение на этих двух резисторах. Предварительно находят падение напряжения на R9.

U9 = R9*I6 = 8*0,02 = 0,16 В.

Теперь напряжение, падающее на Rэкв.7,8, будет разностью между U CD и U9.

U7,8 = UCD – U9= 0,17 – 0,16 = 1 В.

После этого можно уже узнать значение токов, движущихся по резисторам R7 и R8, используя формулы:

  • I4 = U7,8/R7 = 1/2 = 0,5 A;
  • I5 = U7,8/R8 = 1/2 = 0,5 A.

Стоит заметить! Ток, протекающий через R4 и R5, по своему значению равен току на отрезке, не имеющем разветвления.

Рассчитывая схемы и решая задачи по нахождению значений электрических параметров, необходимо использовать эквивалентные сопротивления. С помощью такой замены сложные построения превращаются в элементарные цепи, которые сводятся к параллельным и последовательным соединениям резистивных элементов.

Видео

Общее сопротивление — цепь — Большая Энциклопедия Нефти и Газа, статья, страница 2

Общее сопротивление — цепь

Cтраница 2

Определить общее сопротивление цепи ( рис. 12.69) и напряжение между точками А и В, если амперметр показывает 4 А.
 [16]

Определить общее сопротивление цепи ( рис. 12.69) и напряжение между точками Л и В, если амперметр показывает 4 А.
 [17]

Определить общее сопротивление цепи ( рис. 12.69) и напряжение между точками А и В, если амперметр показывает 4 А.
 [18]

Определим общее сопротивление цепи R экв.
 [19]

Поэтому общее сопротивление цепи кенотронного выпрямителя снижается, ток выпрямителя возрастает, увеличивается минус в точке а. Таким образом, отрицательное смещение на сетке предварительного усилителя возрастает, а экранное напряжение падает, в результате усилитель запирается.
 [20]

Подсчет общего сопротивления цепи при параллельном соединении большого числа различных сопротивлений довольно сложен.
 [21]

При этом общее сопротивление цепи, представленное кривой Z, будет постоянным, независимо от температуры.
 [23]

Кобщ — общее сопротивление цепи.
 [24]

Для нахождения общего сопротивления цепи следует определить общее сопротивление участка цепи, состоящего из вклю ченных параллельно сопротивлений, а затем сложить значение найденного сопротивления со значениями сопротивлений, включенных последовательно.
 [26]

Для нахождения общего сопротивления цепи следует определить общее сопротивление участка цепи, состоящего из включенных параллельно сопротивлений, а затем сложить значение найденного сопротивления со значениями сопротивлений, включенных последовательно.
 [28]

Определить величину общего сопротивления цепи при одновременном включении двух главных фар ( спирали дальнего света) и заднего номерного фонаря автомобиля ГАЗ-51, если рабочее напряжение 12 в, каждая спираль дальнего света пропускает ток в 4 а и спираль лампы освещения номерного знака 0 4 а.
 [29]

Ом), общее сопротивление цепи настолько уменьшается, что сигнал не регистрируется. Стандартный усилитель обычно дает возможность также переключать полярности и масштабы сигнала.
 [30]

Страницы:  

   1

   2

   3

   4




Расчет сопротивления

36 Физика

Ðèñ. 10 á

Ответ: 6 Ом.

Заметим, что преобразуя цепь к более простой эквивалентной, можно на схемах изгибать, удлинять или укорачивать провода, перемещать узлы вдоль проводов, поскольку провода, соединяющие элементы схемы, считаются идеальными, т. е. имеющими нулевое сопротивление.

Рассмотрим теперь, как может видоизмениться цепь, состоящая из тр¸х последовательно соедин¸нных проводников, если к ним добавить другие проводники. Сначала добавим два проводника с нулевым сопротивлением, соединив ими точку А с точ- кой С, а В – с D (рис. 11).

Задача 3. Найдите сопротивление между точками А и D цепи (рис. 11), если каждый проводник имеет сопротивление 3 Ом.

Ðèñ. 11

Решение. Так как точки А и С соединены проводом с нулевым сопротивлением, то их потенциалы равны, а значит, эти точки имеют одинаковый потенциал, и их можно собрать в один общий узел. Аналогично поступим с точками В и D. Таким образом, полу- чилось, что каждый проводник оказался подключенным к одним и тем же точкам, т. е. все три проводника соединены параллельно. Теперь уже легко найти общее сопротивление цепи, учи- тывая, что все проводники имеют одинаковое сопротивление:

1

1

 

1

 

1

3

R

R1

1

Îì.

R

 

R1

R1

R1

 

R1

3

 

 

 

 

 

 

 

Ответ: 1 Ом.

Заменим теперь один из идеальных проводников реальным – с ненулевым сопротивлением.

Задача 4. Определите сопротивление между точками А и D цепи (рис. 12), если каждый проводник имеет сопротивление 6 Ом.

Ðèñ. 12

Заметим, что точки В и D попрежнему соединены проводником с нулевым сопротивлением. Следовательно, они имеют одинаковый потенциал. Соединив их в один узел, получим эквивалентную схему (рис. 13 а). Если же затем «распрямить» е¸, то участки с параллельно и последовательно соедин¸н- ными проводниками станут хорошо видны (рис. 13 б).

Ðèñ. 13 à

Ðèñ. 13 á

После этого останется лишь произвести расч¸т сопротивления для эквивалентной схемы в три этапа:

Как рассчитать резисторы, включенные последовательно и параллельно — Kitronik Ltd

Резисторы серии

Когда резисторы подключаются друг за другом, это называется последовательным соединением. Это показано ниже. Чтобы рассчитать общее общее сопротивление ряда резисторов, подключенных таким образом, вы складываете отдельные сопротивления. Это делается по следующей формуле: Rtotal = R1 + R2 + R3 и так далее. Пример: чтобы рассчитать общее сопротивление для этих трех последовательно соединенных резисторов.

Rtotal = R1 + R2 + R3 = 100 + 82 + 1 Ом = 183 Ом

Задача 1:

Рассчитайте общее сопротивление следующего последовательно включенного резистора.

R Итого = _______________
= _______________
R Итого = _______________
= _______________
R Итого = _______________
= _______________

Параллельные резисторы

Когда резисторы подключаются друг к другу (бок о бок), это называется параллельным подключением. Это показано ниже.

Два параллельных резистора

Чтобы рассчитать общее полное сопротивление двух резисторов, подключенных таким образом, вы можете использовать следующую формулу:

Пример: чтобы рассчитать полное сопротивление для этих двух резисторов, включенных параллельно.


Задача 2:

Рассчитайте общее сопротивление следующего резистора, включенного параллельно.

Три или более резистора параллельно

Для расчета общего общего сопротивления ряда из трех или более резисторов, подключенных таким образом, вы можете использовать следующую формулу: Пример: Для расчета общего сопротивления для этих трех резисторов, включенных параллельно


Задача 3:

Рассчитайте общее сопротивление следующего резистора, включенного параллельно.


Ответы

Задача 1

1 = 1492 Ом 2 = 2242 Ом 3 = 4847 Ом

Задача 2

1 = 5 Ом 2 = 9,57 Ом 3 = 248,12 Ом

Задача 3

1 = 5,95 Ом 2 = 23,76 Ом Загрузите pdf-версию этой страницы здесь. Узнать больше об авторе подробнее »

© Kitronik Ltd — Вы можете распечатать эту страницу и дать ссылку на нее, но не должны копировать страницу или ее часть без предварительного письменного согласия Kitronik.

резисторов последовательно и параллельно

Резисторы серии

Общее сопротивление в цепи с последовательно включенными резисторами равно сумме отдельных сопротивлений.

Цели обучения

Рассчитайте общее сопротивление в цепи с последовательно включенными резисторами

Основные выводы

Ключевые моменты
  • Одинаковый ток течет последовательно через каждый резистор.
  • Отдельные последовательно включенные резисторы не получают полное напряжение источника, а делят его.
  • Общее сопротивление в последовательной цепи равно сумме отдельных сопротивлений: [латекс] \ text {RN} (\ text {series}) = \ text {R} _1 + \ text {R} _2 + \ text {R} _3 +… + \ text {R} _ \ text {N} [/ latex].
Ключевые термины
  • серия : ряд вещей, которые следуют одна за другой или связаны друг за другом.
  • сопротивление : Противодействие прохождению электрического тока через этот элемент.

Обзор

В большинстве схем имеется более одного компонента, называемого резистором, который ограничивает поток заряда в цепи. Мера этого предела расхода заряда называется сопротивлением. Самыми простыми комбинациями резисторов являются последовательное и параллельное соединение.Общее сопротивление комбинации резисторов зависит как от их индивидуальных значений, так и от того, как они подключены.

Последовательные цепи : Краткое введение в анализ последовательных и последовательных цепей, включая закон Кирхгофа по току (KCL) и закон Кирхгофа по напряжению (KVL).

Резисторы

серии

Резисторы

включены последовательно, когда заряд или ток должны проходить через компоненты последовательно.

Резисторы в серии : Эти четыре резистора подключены последовательно, потому что, если бы ток подавался на один конец, он бы протекал через каждый резистор последовательно до конца.

показывает резисторы, последовательно подключенные к источнику напряжения. Общее сопротивление в цепи равно сумме отдельных сопротивлений, так как ток должен последовательно проходить через каждый резистор.

Резисторы, подключенные последовательно : три резистора, подключенные последовательно к батарее (слева), и эквивалентное одиночное или последовательное сопротивление (справа).

Использование закона Ома для расчета изменений напряжения в резисторах серии

В соответствии с законом Ома падение напряжения V на резисторе при протекании через него тока рассчитывается по формуле V = IR, где I — ток в амперах (A), а R — сопротивление в омах (Ω). .

Таким образом, падение напряжения на R 1 равно V 1 = IR 1 , на R 2 равно V 2 = IR 2 , а на R 3 равно V 3 = IR 3 .Сумма напряжений будет равна: V = V 1 + V 2 + V 3 , исходя из сохранения энергии и заряда. Если подставить значения отдельных напряжений, получим:

[латекс] \ text {V} = \ text {IR} _1 + \ text {IR} _2 + \ text {IR} _3 [/ latex]

или

[латекс] \ text {V} = \ text {I} (\ text {R} _1 + \ text {R} _2 + \ text {R} _3) [/ латекс]

Это означает, что полное сопротивление в серии равно сумме отдельных сопротивлений. Следовательно, для каждой цепи с Н количество резисторов, включенных последовательно:

[латекс] \ text {RN} (\ text {series}) = \ text {R} _1 + \ text {R} _2 + \ text {R} _3 +… + \ text {R} _ \ text {N }.[/ латекс]

Поскольку весь ток должен проходить через каждый резистор, он испытывает сопротивление каждого, и последовательно соединенные сопротивления просто складываются.

Поскольку напряжение и сопротивление имеют обратную зависимость, отдельные последовательно включенные резисторы не получают полное напряжение источника, а делят его. Об этом свидетельствует пример, когда две лампочки соединены в последовательную цепь с аккумулятором. В простой схеме, состоящей из одной батареи 1,5 В и одной лампочки, падение напряжения на лампе равно 1.5V через него. Однако, если две лампочки были соединены последовательно с одной и той же батареей, на каждой из них было бы падение напряжения 1,5 В / 2 или 0,75 В. Это будет очевидно по яркости света: каждая из двух последовательно соединенных лампочек будет вдвое слабее, чем одиночная лампочка. Следовательно, резисторы, подключенные последовательно, потребляют такое же количество энергии, как и один резистор, но эта энергия распределяется между резисторами в зависимости от их сопротивлений.

Параллельные резисторы

Общее сопротивление в параллельной цепи равно сумме обратных сопротивлений каждого отдельного сопротивления.

Цели обучения

Рассчитайте полное сопротивление в цепи с резисторами, включенными параллельно

Основные выводы

Ключевые моменты
  • Общее сопротивление в параллельной цепи меньше наименьшего из отдельных сопротивлений.
  • Каждый резистор, включенный параллельно, имеет то же напряжение, что и приложенный к нему источник (напряжение в параллельной цепи постоянно).
  • Параллельные резисторы не получают общий ток каждый; они делят его (ток зависит от номинала каждого резистора и общего количества резисторов в цепи).
Ключевые термины
  • сопротивление : Противодействие прохождению электрического тока через этот элемент.
  • параллельно : расположение электрических компонентов, при котором ток течет по двум или более путям.

Обзор

Резисторы в цепи могут быть включены последовательно или параллельно. Общее сопротивление комбинации резисторов зависит как от их индивидуальных значений, так и от того, как они подключены.

Параллельные схемы : Краткий обзор анализа параллельных цепей с использованием таблиц VIRP для студентов-физиков средней школы.

Параллельные резисторы

Резисторы

включены параллельно, когда каждый резистор подключен непосредственно к источнику напряжения путем соединения проводов с незначительным сопротивлением. Таким образом, к каждому резистору приложено полное напряжение источника.

Параллельное соединение резисторов : Параллельное соединение резисторов.

Каждый резистор потребляет такой же ток, как если бы он был единственным резистором, подключенным к источнику напряжения. Это верно для схем в доме или квартире. Каждая розетка, подключенная к прибору («резистор»), может работать независимо, и ток не должен проходить через каждое устройство последовательно.

Закон и параллельные резисторы

На каждый резистор в цепи подается полное напряжение. Согласно закону Ома, токи, протекающие через отдельные резисторы, равны [латекс] \ text {I} _1 = \ frac {\ text {V}} {\ text {R} _1} [/ latex], [latex] \ text {I} _2 = \ frac {\ text {V}} {\ text {R} _2} [/ latex] и [latex] \ text {I} _3 = \ frac {\ text {V}} {\ text {R} _3} [/ латекс].Сохранение заряда подразумевает, что полный ток является суммой этих токов: 90 · 109

Параллельные резисторы : три резистора, подключенные параллельно батарее, и эквивалентное одиночное или параллельное сопротивление.

[латекс] \ text {I} = \ text {I} _1 + \ text {I} _2 + \ text {I} _3. [/ Latex]

Подстановка выражений для отдельных токов дает:

[латекс] \ text {I} = \ frac {\ text {V}} {\ text {R} _1} + \ frac {\ text {V}} {\ text {R} _2} + \ frac {\ текст {V}} {\ text {R} _3} [/ latex]

или

[латекс] \ text {I} = \ text {V} (\ frac {1} {\ text {R} _1} + \ frac {1} {\ text {R} _2} + \ frac {1} { \ text {R} _3}) [/ latex]

Это означает, что полное сопротивление в параллельной цепи равно сумме обратных сопротивлений каждого отдельного сопротивления. Следовательно, для каждой схемы с числом [latex] \ text {n} [/ latex] или параллельно подключенных резисторов

[латекс] \ text {R} _ {\ text {n} \; (\ text {parallel})} = \ frac {1} {\ text {R} _1} + \ frac {1} {\ text { R} _2} + \ frac {1} {\ text {R} _3}… + \ frac {1} {\ text {R} _ \ text {n}}. [/ Latex]

Это соотношение приводит к общему сопротивлению, которое меньше наименьшего из отдельных сопротивлений. Когда резисторы подключены параллельно, от источника течет больше тока, чем протекает для любого из них по отдельности, поэтому общее сопротивление ниже.

Каждый резистор, включенный параллельно, имеет такое же полное напряжение источника, как на него, но делит общий ток между ними. Примером может служить соединение двух лампочек в параллельную цепь с аккумулятором на 1,5 В. В последовательной цепи две лампочки будут вдвое менее тусклыми при подключении к одному источнику батареи. Однако, если бы две лампочки были подключены параллельно, они были бы столь же яркими, как если бы они были подключены к батарее по отдельности. Поскольку к обеим лампочкам подается одинаковое полное напряжение, батарея также разряжается быстрее, поскольку она, по сути, поставляет полную энергию обеим лампочкам.В последовательной цепи батарея будет работать столько же, сколько и с одной лампочкой, только тогда яркость распределяется между лампочками.

Комбинированные схемы

Комбинированная схема может быть разбита на аналогичные части, которые работают последовательно или параллельно.

Цели обучения

Описать расположение резисторов в комбинированной схеме и его практическое значение

Основные выводы

Ключевые моменты
  • Более сложные соединения резисторов иногда представляют собой просто комбинации последовательного и параллельного.
  • Различные части комбинированной схемы могут быть идентифицированы как последовательные или параллельные, уменьшены до их эквивалентов, а затем уменьшены до тех пор, пока не останется единственное сопротивление.
  • Сопротивление в проводах снижает ток и мощность, подаваемые на резистор. Если сопротивление в проводах относительно велико, как в изношенном (или очень длинном) удлинителе, то эти потери могут быть значительными и повлиять на выходную мощность в бытовые приборы.
Ключевые термины
  • серия : ряд вещей, которые следуют одна за другой или связаны друг за другом.
  • параллельно : расположение электрических компонентов, при котором ток течет по двум или более путям.
  • Комбинированная схема : электрическая цепь, содержащая несколько резисторов, соединенных комбинацией последовательного и параллельного соединения.

Комбинированные схемы

Более сложные соединения резисторов иногда представляют собой просто комбинации последовательного и параллельного. Это часто встречается, особенно если учитывать сопротивление проводов.В этом случае сопротивление провода включено последовательно с другими сопротивлениями, включенными параллельно.

Комбинированная цепь может быть разбита на аналогичные части, которые являются последовательными или параллельными, как показано на схеме. На рисунке общее сопротивление может быть вычислено путем соединения трех резисторов друг с другом последовательно или параллельно. R 1 и R 2 соединены параллельно по отношению друг к другу, поэтому мы знаем, что для этого подмножества сопротивление, обратное сопротивлению, будет равно:

Сеть резисторов : В этой комбинированной схеме цепь может быть разбита на последовательный компонент и параллельный компонент.

Комбинированные схемы : два параллельных резистора, соединенные последовательно с одним резистором.

[латекс] \ frac {1} {\ text {R} _1} + \ frac {1} {\ text {R} _2} [/ latex] или [латекс] \ frac {\ text {R} _1 \ text {R} _2} {\ text {R} _1 + \ text {R} _2} [/ latex]

R 3 соединен последовательно с и R 1 и R 2 , поэтому сопротивление будет рассчитываться как:

[латекс] \ text {R} = \ frac {\ text {R} _1 \ text {R} _2} {\ text {R} _1 + \ text {R} _2} + \ text {R} _3 [/ latex ]

Сложные комбинированные схемы

Для более сложных комбинированных схем различные части могут быть идентифицированы как последовательные или параллельные, уменьшены до их эквивалентов, а затем уменьшены до тех пор, пока не останется одно сопротивление, как показано на. На этом рисунке комбинация из семи резисторов идентифицирована как включенные последовательно или параллельно. На исходном изображении две обведенные кружком секции показывают резисторы, включенные параллельно.

Сокращение комбинированной схемы : Эта комбинация из семи резисторов имеет как последовательные, так и параллельные части. Каждый из них идентифицируется и уменьшается до эквивалентного сопротивления, а затем уменьшается до тех пор, пока не будет достигнуто одно эквивалентное сопротивление.

Уменьшение этих параллельных резисторов до одного значения R позволяет нам визуализировать схему в более упрощенном виде.На верхнем правом изображении мы видим, что обведенная кружком часть содержит два последовательно соединенных резистора. Мы можем дополнительно уменьшить это до другого значения R, добавив их. Следующий шаг показывает, что два обведенных резистора включены параллельно. Уменьшение тех ярких моментов, что последние два включены последовательно и, таким образом, могут быть уменьшены до одного значения сопротивления для всей цепи.

Практическое применение комбинированной схемы состоит в том, что сопротивление в проводах снижает ток и мощность, подаваемую на резистор.Комбинированная цепь может быть преобразована в последовательную цепь на основе понимания эквивалентного сопротивления параллельных ветвей комбинированной цепи. Последовательная цепь может использоваться для определения общего сопротивления цепи. По сути, сопротивление провода является последовательным с резистором. Таким образом, увеличивается общее сопротивление и уменьшается ток. Если сопротивление провода относительно велико, как в изношенном (или очень длинном) удлинителе, то эти потери могут быть значительными. Если потребляется большой ток, падение ИК-излучения в проводах также может быть значительным.

Зарядка аккумулятора: последовательные и параллельные ЭДС

При последовательном включении источников напряжения их ЭДС и внутренние сопротивления складываются; параллельно они остаются прежними.

Цели обучения

Сравните сопротивления и электродвижущие силы для источников напряжения, подключенных с одинаковой и противоположной полярностью, последовательно и параллельно

Основные выводы

Ключевые моменты
  • ЭДС, соединенные последовательно с одинаковой полярностью, являются аддитивными и приводят к более высокой полной ЭДС.
  • Две ЭДС, соединенные последовательно с противоположной полярностью, имеют общую ЭДС, равную разнице между ними, и могут использоваться для зарядки источника более низкого напряжения.
  • Два источника напряжения с идентичными ЭДС, соединенные параллельно, имеют чистую ЭДС, эквивалентную одному источнику ЭДС, однако общее внутреннее сопротивление меньше, и, следовательно, вырабатывается более высокий ток.
Ключевые термины
  • параллельно : расположение электрических компонентов, при котором ток течет по двум или более путям.
  • электродвижущая сила : (ЭДС) — напряжение, генерируемое батареей или магнитной силой в соответствии с законом Фарадея. Она измеряется в вольтах, а не в ньютонах, и поэтому на самом деле не является силой.
  • серия : ряд вещей, которые следуют одна за другой или связаны друг за другом.

Когда используется более одного источника напряжения, они могут быть подключены последовательно или параллельно, подобно резисторам в цепи. Когда источники напряжения включены последовательно в одном направлении, их внутренние сопротивления складываются, а их электродвижущая сила, или ЭДС, складывается алгебраически. Эти типы источников напряжения распространены в фонариках, игрушках и других устройствах. Обычно ячейки включены последовательно, чтобы обеспечить большую суммарную ЭДС.

Фонарик и лампочка : Последовательное соединение двух источников напряжения в одном направлении. Эта схема представляет собой фонарик с двумя последовательно включенными ячейками (источниками напряжения) и одной лампочкой (сопротивление нагрузки).

Батарея — это соединение нескольких гальванических элементов. Однако недостатком такого последовательного соединения ячеек является то, что их внутреннее сопротивление увеличивается. Иногда это может быть проблематично. Например, если вы поместите в машину две батареи на 6 В вместо обычной батареи на 12 В, вы должны добавить как ЭДС, так и внутреннее сопротивление каждой батареи. Таким образом, вы получите ту же ЭДС 12 В, хотя внутреннее сопротивление тогда будет удвоено, что вызовет у вас проблемы, когда вы захотите запустить двигатель.

Но если ячейки противостоят друг другу — например, когда одна вставляется в прибор задом наперед, — общая ЭДС меньше, поскольку она представляет собой алгебраическую сумму отдельных ЭДС. Когда он перевернут, он создает ЭДС, которая противодействует другой, и приводит к разнице между двумя источниками напряжения.

Зарядное устройство для аккумулятора : представляет два источника напряжения, соединенных последовательно с противоположными ЭДС. Ток течет в направлении большей ЭДС и ограничивается суммой внутренних сопротивлений.(Обратите внимание, что каждая ЭДС представлена ​​на рисунке буквой E.) Зарядное устройство, подключенное к аккумулятору, является примером такого подключения. Зарядное устройство должно иметь большую ЭДС, чем аккумулятор, чтобы через него протекал обратный ток.

Когда два источника напряжения с идентичными ЭДС соединены параллельно и также подключены к сопротивлению нагрузки, общая ЭДС равна индивидуальной ЭДС. Но общее внутреннее сопротивление уменьшается, поскольку внутренние сопротивления параллельны. Таким образом, параллельное соединение может производить больший ток.

Два идентичных ЭДС : Два источника напряжения с одинаковыми ЭДС (каждый помечен буквой E), подключенные параллельно, создают одинаковую ЭДС, но имеют меньшее общее внутреннее сопротивление, чем отдельные источники. Параллельные комбинации часто используются для подачи большего тока.

ЭДС и напряжение на клеммах

Выходное напряжение или напряжение на клеммах источника напряжения, такого как аккумулятор, зависит от его электродвижущей силы и внутреннего сопротивления.

Цели обучения

Выразите взаимосвязь между электродвижущей силой и напряжением на клеммах в форме уравнения

Основные выводы

Ключевые моменты
  • Электродвижущая сила (ЭДС) — это разность потенциалов источника при отсутствии тока.
  • Напряжение на клеммах — это выходное напряжение устройства, измеренное на его клеммах.
  • Напряжение на клеммах рассчитывается по формуле V = ЭДС — Ir.
Ключевые термины
  • электродвижущая сила : (ЭДС) — напряжение, генерируемое батареей или магнитной силой в соответствии с законом Фарадея. Она измеряется в вольтах, а не в ньютонах, и поэтому на самом деле не является силой.
  • напряжение на клеммах : выходное напряжение устройства, измеренное на его клеммах.
  • разность потенциалов : разница в потенциальной энергии между двумя точками в электрическом поле; разница в заряде между двумя точками в электрической цепи; Напряжение.

Когда вы забываете выключить автомобильные фары, они постепенно тускнеют по мере разряда аккумулятора. Почему они просто не мигают, когда батарея разряжена? Их постепенное затемнение означает, что выходное напряжение батареи уменьшается по мере разряда батареи. Причина снижения выходного напряжения для разряженных или перегруженных батарей заключается в том, что все источники напряжения состоят из двух основных частей — источника электрической энергии и внутреннего сопротивления.

Электродвижущая сила

Все источники напряжения создают разность потенциалов и могут подавать ток, если подключены к сопротивлению. В небольшом масштабе разность потенциалов создает электрическое поле, которое воздействует на заряды, вызывая ток. Мы называем эту разность потенциалов электродвижущей силой (сокращенно ЭДС). ЭДС — это вообще не сила; это особый тип разности потенциалов источника при отсутствии тока. Единицы измерения ЭДС — вольт.

Электродвижущая сила напрямую связана с источником разности потенциалов, например с конкретной комбинацией химических веществ в батарее.Однако при протекании тока ЭДС отличается от выходного напряжения устройства. Напряжение на выводах батареи, например, меньше, чем ЭДС, когда батарея подает ток, и оно уменьшается дальше, когда батарея разряжается или разряжается. Однако, если выходное напряжение устройства можно измерить без потребления тока, то выходное напряжение будет равно ЭДС (даже для сильно разряженной батареи).

Напряжение на клеммах

представляет схематическое изображение источника напряжения.Выходное напряжение устройства измеряется на его клеммах и называется напряжением на клеммах В . Напряжение на клеммах определяется уравнением:

Схематическое изображение источника напряжения : Любой источник напряжения (в данном случае углеродно-цинковый сухой элемент) имеет ЭДС, связанную с источником разности потенциалов, и внутреннее сопротивление r, связанное с его конструкцией. (Обратите внимание, что сценарий E означает ЭДС.) Также показаны выходные клеммы, на которых измеряется напряжение на клеммах V.Поскольку V = ЭДС-Ir, напряжение на клеммах равно ЭДС, только если ток не течет.

[латекс] \ text {V} = \ text {emf} — \ text {Ir} [/ latex],

где r — внутреннее сопротивление, а I — ток, протекающий во время измерения.

I является положительным, если ток течет от положительного вывода. Чем больше ток, тем меньше напряжение на клеммах. Точно так же верно, что чем больше внутреннее сопротивление, тем меньше напряжение на клеммах.

Расчет общего сопротивления — Физика средней школы

Если вы считаете, что контент, доступный через Веб-сайт (как определено в наших Условиях обслуживания), нарушает
или больше ваших авторских прав, сообщите нам, отправив письменное уведомление («Уведомление о нарушении»), содержащее
то
информацию, описанную ниже, назначенному ниже агенту.Если репетиторы вуза предпримут действия в ответ на
ан
Уведомление о нарушении, оно предпримет добросовестную попытку связаться со стороной, которая предоставила такой контент
средствами самого последнего адреса электронной почты, если таковой имеется, предоставленного такой стороной Varsity Tutors.

Ваше Уведомление о нарушении прав может быть отправлено стороне, предоставившей доступ к контенту, или третьим лицам, таким как
так как
ChillingEffects. org.

Обратите внимание, что вы будете нести ответственность за ущерб (включая расходы и гонорары адвокатов), если вы
искажать информацию о том, что продукт или действие нарушает ваши авторские права.Таким образом, если вы не уверены, что контент находится
на Веб-сайте или по ссылке с него нарушает ваши авторские права, вам следует сначала обратиться к юристу.

Выполните следующие действия, чтобы отправить уведомление:

Вы должны включить следующее:

Физическая или электронная подпись владельца авторских прав или лица, уполномоченного действовать от их имени;
Идентификация авторских прав, которые, как утверждается, были нарушены;
Описание характера и точного расположения контента, который, по вашему мнению, нарушает ваши авторские права, в \
достаточно подробностей, чтобы позволить репетиторам университетских школ найти и точно идентифицировать этот контент; например нам требуется
а
ссылка на конкретный вопрос (а не только на название вопроса), который содержит содержание и описание
к какой конкретной части вопроса — изображению, ссылке, тексту и т. д. — относится ваша жалоба;
Ваше имя, адрес, номер телефона и адрес электронной почты; и
Ваше заявление: (а) вы добросовестно считаете, что использование контента, который, по вашему мнению, нарушает
ваши авторские права не разрешены законом, владельцем авторских прав или его агентом; (б) что все
информация, содержащаяся в вашем Уведомлении о нарушении прав, является точной, и (c) под страхом наказания за лжесвидетельство, что вы
либо владелец авторских прав, либо лицо, уполномоченное действовать от их имени.

Отправьте жалобу нашему уполномоченному агенту по адресу:

Чарльз Кон
Varsity Tutors LLC
101 S. Hanley Rd, Suite 300
St. Louis, MO 63105

Или заполните форму ниже:

последовательно соединенных резисторов — Расчет сопротивления — CCEA — Редакция GCSE Physics (Single Science) — CCEA

g65m7v2a1c.0.0.0.1:0.1.0.$0.$1.$0″> Ток

При последовательном подключении резисторов ток через каждый резистор одинаков.

Ток одинаков во всех точках последовательной цепи.

В схеме ниже: I S = I 1 = I 2 = I 3

Напряжение В (или разность потенциалов)

При последовательном соединении резисторов общая сумма напряжение (иногда называемое разностью потенциалов) на каждом компоненте равно напряжению на источнике питания.

g65m7v2a1c.0.0.0.1:0.1.0.$0.$1.$8″> В приведенной выше схеме:

V S = V 1 + V 2 + V 3

Это просто форма закона сохранения энергии .

Напряжение питания — это мера энергии, подводимой к каждому электрону.

Напряжение на каждом компоненте — это электрическая энергия, преобразованная каждым компонентом.

Следовательно, поданная энергия равна преобразованной энергии — энергия не была создана или разрушена в цепи.

В последовательной цепи напряжение на источнике питания равно сумме напряжений на каждом компоненте.

Сопротивление

Общее сопротивление R двух или более резисторов, соединенных последовательно, является суммой отдельных сопротивлений резисторов.

Для схемы выше общее сопротивление R определяется по формуле:

R = R 1 + R 2 + R 3

Пример

Найдите полное сопротивление цепи выше.

Ответ

g65m7v2a1c.0.0.0.1:0.1.0.$0.$4.$1″> Это последовательная цепь, поэтому полное сопротивление определяется по формуле:

R = R 1 + R 2 + R 3 + R 4

R = \ ({ 4} \ Omega + {8} \ Omega + {2} \ Omega + {12} \ Omega \)

R = \ ({26} \ Omega \)

Полное сопротивление цепи резисторов равно \ ( {26} \ Omega \).Это означает, что четыре отдельных резистора можно заменить одним резистором из \ ({26} \ Omega \).

Последовательное добавление резисторов всегда увеличивает общее сопротивление.

Ток должен проходить через каждый резистор по очереди, поэтому добавление дополнительного резистора увеличивает уже встреченное сопротивление.

Параллельные резисторы

Ток

При параллельном подключении резисторов ток от источника питания равен сумме токов, протекающих через каждую ветвь цепи.

Другими словами, токи в ветвях параллельной цепи складываются в ток питания.

В приведенной выше схеме:

I S = I 1 + I $0.$6.$4.$5″> 2 + I 3

Это соотношение выражает закон сохранения заряда.

Все электроны, вышедшие из источника питания, должны вернуться в источник, и каждый электрон может пройти только через одну параллельную ветвь.

В параллельной цепи ток от источника питания равен сумме токов в каждой ветви цепи.

Напряжение

В параллельной цепи напряжение на каждой ветви цепи равно напряжению питания.

Для схемы выше:

В S = V 1 = V g65m7v2a1c.0.0.0.1:0.1.0.$0.$7.$3.$5″> 2 = V 3

В параллельной цепи напряжение на каждой ветви равно напряжению питания.

Сопротивление

При параллельном подключении резисторов общее сопротивление R рассчитывается по формуле:

\ [\ frac {1} {R} = \ frac {1} {R} _ {1} + \ frac {1} {R} _ {2} + \ frac {1} {R} _ {3} \]

Серия

и параллельные резисторы

  • Изучив этот раздел, вы сможете:
  • Рассчитайте значения общего сопротивления в цепях с последовательным сопротивлением.
  • Используйте соответствующие формулы для расчета сопротивления в цепях с параллельным сопротивлением.
  • • Вычисление суммы обратных величин.
  • • Произведение над суммой.
  • Рассчитайте значения общего сопротивления в последовательно / параллельных сетях.

Расчеты в последовательно- и параллельных резисторных цепях

Компоненты, включая резисторы в цепи, могут быть соединены вместе двумя способами:

ПОСЛЕДОВАТЕЛЬНО, так что один и тот же ток течет через все компоненты, но на каждом из них может существовать разная разность потенциалов (напряжение).

ПАРАЛЛЕЛЬНО, так что одна и та же разность потенциалов (напряжение) существует на всех компонентах, но каждый компонент может проводить разный ток.

Рис. 4.2.1 Резисторы серии

Рис. 4.2.2 Параллельные резисторы

В любом случае (для резисторов) общее сопротивление той части цепи, которая содержит резисторы, может быть рассчитано с использованием методов, описанных ниже.

Возможность рассчитать суммарное (общее) значение резисторов таким способом позволяет легко вычислить неизвестные значения сопротивления, тока и напряжения для довольно сложных схем с использованием относительно простых методов. Это очень полезно при поиске неисправностей.

ПЕРЕД ДАЛЬНЕЙШЕЙ ДАЛЬНОСТЬЮ ПОПРОБУЙТЕ ИСПОЛЬЗОВАНИЕ ФОРМУЛ ДЛЯ РАСЧЕТА ОБЩИХ ЗНАЧЕНИЙ СЕРИЙНЫХ И ПАРАЛЛЕЛЬНЫХ РЕЗИСТОРОВ.

Для резисторов в серии:

Общее сопротивление двух или более резисторов, соединенных последовательно , определяется простым сложением индивидуальных значений резисторов, чтобы найти общую сумму (R TOT ):

Для резисторов, включенных параллельно:

Для расчета общего сопротивления цепи, в которой используются параллельные резисторы, можно использовать следующую формулу.

Обратите внимание, однако, что эта формула НЕ дает вам общего сопротивления R TOT .
Это дает вам ВЗАИМОДЕЙСТВИЕ R TOT или:

Это совсем другое значение — и НЕ является полным сопротивлением. Он делится на 1, деленный на R TOT . Чтобы получить правильное значение для R TOT (которое будет обратным 1 / R TOT , т. Е. TOT /1, просто нажмите соответствующую клавишу на вашем калькуляторе (отмеченную 1 / x или x-1) .

Другой способ расчета параллельных цепей.

Суммарное сопротивление двух резисторов, включенных параллельно , которое не включает обратные, определяется по формуле:

Эту формулу часто называют «произведение над суммой».

Он рассчитывает только ДВА резистора параллельно? Ну да, но это не большая проблема. Если имеется более двух параллельных резисторов, просто выберите два из них и определите общее сопротивление для этих двух — затем используйте это общее сопротивление, как если бы это был один резистор, и составьте еще одну пару с третьим резистором.Определите новое общее значение и так далее, пока вы не включите все параллельные резисторы в этой конкретной сети.

О, еще одна вещь, которую следует помнить о произведении над суммой, видите скобки вокруг суммы (нижняя часть) формулы? Это означает, что вы должны решить это ДО того, как использовать его для разделения продукта (верхняя часть) на. Если вы этого не сделаете, ваш ответ будет неправильным.

Звучит сложно? Не совсем, это просто вопрос повторения, и на практике вы не часто встречаетесь с множеством параллельных сетей с гораздо более чем двумя резисторами.Тем не менее, какую формулу вы выберете, зависит от вас, взаимная или сумма продукта.

Подсказки

Использование обратного метода

Если вы используете МЕТОД ВЗАИМОДЕЙСТВИЯ для параллельных цепей, НЕ ЗАБУДЬТЕ, когда вы добавили обратные величины отдельных резисторов — вы должны снова найти обратную величину. 1 / R1 + 1 / R2 + 1 / R3 = 1 / R TOT , а чтобы найти R TOT , вы должны найти обратную величину 1 / R TOT .

Упрощающие схемы

Для комбинированных последовательных и параллельных цепей сначала разработайте участок цепи (последовательный или параллельный).Затем перерисуйте схему, заменив участок, сопротивление которого вы нашли, одним резистором. Теперь у вас есть упрощенная схема, по которой можно найти R TOT .

Вы можете использовать формулу «произведение на сумму»:

Для цепей с более чем двумя параллельными резисторами просто определите два параллельных резистора одновременно, используя формулу произведения на сумму, а затем перерисуйте схему, заменив два резистора одним резистором, значение которого является объединенным сопротивлением двух .

Теперь вы можете использовать ваше первое комбинированное значение в качестве единственного резистора со следующим параллельным резистором и так далее. Таким образом, можно выработать большое количество параллельных резисторов с использованием произведения на сумму.

Когда все параллельные резисторы одинакового номинала.

Если несколько одинаковых параллельных резисторов подключены, общее сопротивление будет равно номиналу резистора, умноженному на обратную величину количества резисторов.

, то есть два параллельных резистора 12 кОм имеют суммарное сопротивление

12K x 1/2 = 6K

Три параллельно включенных резистора 12 кОм имеют общее сопротивление

12K x 1/3 = 4K и т. Д.

Проверяю ответ

Суммарное значение любого количества параллельных резисторов всегда будет МЕНЬШЕ, чем значение наименьшего отдельного резистора в сети. Используйте этот факт, чтобы проверить свои ответы.

Серия

и параллельная комбинация

Попробуйте выполнить несколько расчетов для последовательной и параллельной цепей резисторов. Для этого вам просто нужно использовать информацию на этой странице и на странице «Советы по расчету резисторов». Вас просят вычислить полное сопротивление для каждой цепи.Вы можете выбрать, какую формулу использовать

Вы также можете получить помощь по математике, загрузив нашу бесплатную брошюру «Советы по математике».

Прежде чем начать, подумайте об этих нескольких советах. Они упростят задачу, если вы будете внимательно им следовать.

1. Разработайте ответы с помощью карандаша и бумаги; перерисуйте схему, над которой работаете.

2. Конечно, ответ будет не просто числом, это будет определенное количество Ом, не забудьте указать правильную единицу (например,г. Ω, KΩ или MΩ), иначе ваш ответ не имеет смысла.

3. Когда вы вводите значения в калькулятор, преобразуйте все значения KΩ или MΩ в Ом с помощью клавиши EXP. Если вы здесь ошибетесь, то получите действительно глупые ответы, в тысячи раз слишком большие или слишком маленькие.

Итак, вы прочитали эти инструкции и готовы приступить к работе. Вот способ решить типичную проблему на бумаге, чтобы (с практикой) вы не запутались.

Пример последовательной и параллельной цепей

.

Хорошо, здесь есть что вспомнить, так почему бы не попробовать несколько практических вопросов в Resistors Module 4.5 по определению полного сопротивления некоторых цепей резисторов?

Основная электрическая теория в двух словах

теория


от
electrician2.com



Правила цепей постоянного тока серии

Правило № 1:

Одинаковый ток течет через каждую часть серии
цепь.

Правило № 2:

Общее сопротивление последовательной цепи равно
сумма отдельных сопротивлений.

Правило № 3:

Общее напряжение в последовательной цепи равно
сумме индивидуальных падений напряжения.

Правило № 4:

Падение напряжения на резисторе в последовательной цепи
пропорционально размеру резистора.

Правило № 5:

Общая мощность, рассеиваемая в последовательной цепи, составляет
равняется сумме индивидуальных потерь мощности.

РЕЗЮМЕ ФОРМУЛ ЗАКОНА OHMS

АМПЕР =

ВОЛЬТ

СОПРОТИВЛЕНИЕ

СОПРОТИВЛЕНИЕ =

ВОЛЬТ

АМПЕР

ВОЛЬТ =

АМПЕР x СОПРОТИВЛЕНИЕ

Параллельная цепь постоянного тока
Правила

Правило № 1:

Такое же напряжение присутствует на каждой ветви
параллельная цепь и равна напряжению источника.

Правило № 2:

Ток через ответвление параллельной сети
обратно пропорционально сопротивлению ветви.

Правило № 3:

Суммарный ток параллельной цепи равен
сумма токов отдельных ветвей цепи.

Правило № 4:

Общее сопротивление параллельной цепи равно
к сумме обратных величин отдельных сопротивлений
схемы.

Правило № 5:

Общая мощность, рассеиваемая в параллельной цепи, составляет
равняется сумме индивидуальных потерь мощности.
СВОДКА ПРАВИЛ ПАРАЛЛЕЛЬНЫХ ЦЕПЕЙ

ОБЩЕЕ НАПРЯЖЕНИЕ =

E (1) = E (2) = E (3) . .. и т. Д.

ИТОГО
СОПРОТИВЛЕНИЕ =

ВОЛЬТ

АМПЕР

ВОЛЬТ
=

ОБЩЕЕ НАПРЯЖЕНИЕ

ОБЩИЙ АМПЕР

К
ОПРЕДЕЛИТЕ ПОЛНОЕ СОПРОТИВЛЕНИЕ В ПАРАЛЛЕЛЬНОЙ ЦЕПИ ПРИ ПОЛНОМ ТОКЕ
И ОБЩЕЕ НАПРЯЖЕНИЕ ПРИ НЕИЗВЕСТНОМ ИСПОЛЬЗОВАНИИ СЛЕДУЮЩЕЙ ФОРМУЛЫ:

RT =


1


___________________

1
+ 1 + 1 +…… и т. д.


R1 R2
R3
ДЛЯ ДВУХ РЕЗИСТОРОВ ПАРАЛЛЕЛЬНО ИСПОЛЬЗУЙТЕ ЭТО
FORMULA НАЗЫВАЕТСЯ «ПРОДУКТ ПРЕВЫШАЮЩИЙ СУММУ»

РТ
=

Р (1)
* Р (2)


р (1) + р (2)

МОЩНОСТЬ В ОДНОЙ ФАЗЕ
РЕЗИСТИВНЫЕ ЦЕПИ


ГДЕ КОЭФФИЦИЕНТ МОЩНОСТИ
100 ПРОЦЕНТОВ


(ЭТИ
ФОРМУЛЫ ОБЫЧНО ИСПОЛЬЗУЮТСЯ ДЛЯ РЕШЕНИЯ БОЛЬШИНСТВА ПРОБЛЕМ С ПИТАНИЕМ ЦЕПЕЙ НА ИСПЫТАНИЯХ)

К
ОПРЕДЕЛЕНИЕ МОЩНОСТИ, ПОТРЕБЛЯЕМОЙ ОТДЕЛЬНЫМ РЕЗИСТОРОМ В СЕРИЙНОЙ ЦЕПИ
ИСПОЛЬЗУЙТЕ ЭТУ ФОРМУЛУ:

К
ОПРЕДЕЛЕНИЕ МОЩНОСТИ, ПОТРЕБЛЯЕМОЙ ОТДЕЛЬНЫМ РЕЗИСТОРОМ В ПАРАЛЛЕЛЬНОЙ ЦЕПИ
ИСПОЛЬЗУЙТЕ ЭТУ ФОРМУЛУ:

К
ОПРЕДЕЛИТЕ ОБЩУЮ МОЩНОСТЬ, ПОТРЕБЛЯЕМУЮ ОТДЕЛЬНОЙ ЦЕПЕЙ, ИСПОЛЬЗУЙТЕ ДАННУЮ ФОРМУЛУ:

МОЩНОСТЬ = E
(ОБЩЕЕ НАПРЯЖЕНИЕ) x I (ОБЩИЙ ТОК)


ПРАВИЛА
ПАЛЬЦА:

  • THE
    ОБЩЕЕ СОПРОТИВЛЕНИЕ РЕЗИСТОРОВ ПО ПАРАЛЛЕЛЬНОМУ ВСЕГДА МЕНЬШЕ ЗНАЧЕНИЯ
    ЛЮБОГО ОДНОГО РЕЗИСТОРА.
  • THE
    ПОЛНОЕ СОПРОТИВЛЕНИЕ ПАРАЛЛЕЛЬНЫХ РЕЗИСТОРОВ, ЧТО ВСЕ ОДИНАКОВЫЕ ЗНАЧЕНИЕ
    ЗНАЧЕНИЕ, ОТДЕЛЕННОЕ НА КОЛИЧЕСТВО РЕЗИСТОРОВ.
  • ВСЕГДА
    ИСПОЛЬЗУЙТЕ ПРЕВЫШАЮЩИЕ ПРАВИЛО ПРОДУКТА, ЧТОБЫ РАЗБИВАТЬ ДВА ПАРАЛЛЕЛЬНЫХ РЕЗИСТОРА
    ОДИН РЕЗИСТОР. ЭТО НАМНОГО ЛЕГЧЕ, ЧЕМ ПОПЫТАТЬСЯ НА РЕШЕНИЕ БОЛЬШОЙ АЛГЕБРАКИ
    ВЫРАЖЕНИЯ.
  • 746
    Ватты равны одной лошади
  • ЭФФЕКТИВНОСТЬ
    РАВНО ВЫХОД, РАЗДЕЛЕННЫЙ НА ВХОД
  • В
    ИНДУКТИВНЫЕ ЦЕПИ, ТОК, ЗАПИСЫВАЕТ НАПРЯЖЕНИЕ.
  • В
    ЕМКОСТНЫЕ ЦЕПИ, ТОК, НАПРЯЖЕНИЕ.
  • СИЛА
    КОЭФФИЦИЕНТ ЯВЛЯЕТСЯ МЕРОПРИЯТИЕМ ДЛЯ ДАЛЬНЕЙШЕГО ТОКА ИЛИ НАПРЯЖЕНИЯ.

ПИТАНИЕ В ПЕРЕМЕННОЙ
ТОКОВЫЕ ЦЕПИ, КОГДА КОЭФФИЦИЕНТ МОЩНОСТИ НЕ 100 ПРОЦЕНТОВ


(Истинная СИЛА)
= E x I x КОЭФФИЦИЕНТ МОЩНОСТИ (ДЛЯ
ОДНА ФАЗА)


(Истинная СИЛА)
= Е х Я х 1. 732 Х КОЭФФИЦИЕНТ МОЩНОСТИ (ДЛЯ
ТРЕХФАЗА)

ЭТО
ВЛАСТЬ ТАКЖЕ НАЗЫВАЕТСЯ ИСТИННОЙ ВЛАСТЬЮ ИЛИ РЕАЛЬНОЙ ВЛАСТЬЮ, ПРОТИВ ВИДНОЙ ВЛАСТИ
НАХОДИТСЯ ПО РАСЧЕТУ НАПРЯЖЕНИЯ.

Ваттметры показывают значение True Power.


Полная мощность = ВОЛЬТ-АМПЕР
= E x I (FOR
ОДНА ФАЗА)


Полная мощность = ВОЛЬТ-АМПЕР
= E x I x 1,732 (ДЛЯ
ТРЕХФАЗА)

IT
МОЖЕТ БЫТЬ ОПРЕДЕЛЕНА АЛГЕБРЫ, ЧТО

КОЭФФИЦИЕНТ МОЩНОСТИ
=


ИСТИННАЯ СИЛА


ВНЕШНЯЯ МОЩНОСТЬ

ДВИГАТЕЛЬ
ФОРМУЛЫ ПРИМЕНЕНИЯ

МОЩНОСТЬ
=


(для трехфазных двигателей)


1. 732 x ВОЛЬТ x АМПЕР x КПД x коэффициент мощности



746

ТРЕХФАЗНЫЙ АМПЕР =

(для трехфазных двигателей)


746 x ПОДСТАВКА


1,732 x ВОЛЬТ x
ЭФФЕКТИВНОСТЬ x КОЭФФИЦИЕНТ МОЩНОСТИ

СИНХРОННЫЙ
Обороты =


ГЕРЦ х 120


КОЛИЧЕСТВО ПОЛЮСОВ

ДВИГАТЕЛЬ
МАРКИРОВКА И ПОДКЛЮЧЕНИЯ


СОЕДИНЕНИЯ
ДЛЯ ДЕВЯТИ СВИНЦА


ТРИ
ФАЗНЫЕ ДВИГАТЕЛИ

ТРИ
ФАЗА ЗВЕЗДА ИЛИ Y

ЗВЕЗДА СОЕДИНЕНА
Напряжение Линия 1 Строка 2 Строка 3 Все вместе
Низкий 1 и 7 2 и 8 3 и 9 4 и 5 и 6
Высоко 1 2 3 4 и 7, 5 и 8, 6 и 9

ТРИ
ФАЗА ДЕЛЬТА

СОЕДИНЕНИЕ ТРЕУГОЛЬНИКОМ
Напряжение Линия 1 Строка 2 Строка 3 Все вместе
Низкий 1 и 6 и 7 2 и 4 и 8 3 и 5 и 9 НИКТО
Высоко 1 2 3 4 и 7, 5 и 8, 6 и 9

ДЕЛЬТА
КЛЮЧ ДЛЯ ТРАНСФОРМАТОРА

ДВИГАТЕЛЬ
КОНТРОЛЛЕР С ТРЕМЯ


ПУСК
ОСТАНОВКИ


(ХОЛДИНГ
КОНТАКТЫ НЕ ПОКАЗАНЫ)

ТРАНСФОРМАТОР
ОБОРОТОВ

Ep = Tp

Es Ts

Где

Ep — первичное напряжение

Es — вторичное напряжение

Tp — количество витков в первичной обмотке

Ts — количество витков вторичной обмотки


Подключение переключателя

4-ходовой

D — S, E — R, F — T, G — W

3-ходовой

от A до Z, от B до Y, от C до X


Пример заполнения кабельного лотка

Решение

Для многожильных кабелей на 2000 В и где
кабели 4/0 и более устанавливаются с кабелями менее 4/0
по размеру см. Раздел 392.22 (А) (1) (с).

Сумма сечений всех кабелей меньшего размера
чем 4/0 не должна превышать максимально допустимую площадь заполнения в результате
расчет в столбце 2 таблицы 392.22 (A) для соответствующего кабельного лотка
ширина.

В этой таблице sd = сумма диаметров кабелей
4/0 и больше.

Тогда

Сумма поперечного сечения для кабелей меньше 4/0
(СУММ) <= X - 1,2 x стандартное отклонение

X определяет ширину кабельного лотка по таблице 392.22 (А).

X> = СУММ + (1,2 x стандартное отклонение)

СУММ = 6 x 3,14 x 0,5 x 0,5 кв. Дюйма

СУММ = 4,71 кв. Дюйма

sd = 3 x 2 дюйма

sd = 6 дюймов

X> = 4,71 + (1,2 x 6)

X> = 11,91

Из таблицы 392.22 (A) следующий больший X равен 14, и это преобразует
к кабельному лотку шириной 12,0 дюймов в колонне 1.

Следует отметить, что кабели 4/0 и больше должны
быть установлен в один слой.


Максимальная мощность

для NEMA-Rated

Пускатели двигателя
Однофазный Трехфазный
NEMA

Размер
115

Вольт
230

Вольт
208/230

Вольт
460/575

Вольт
00 1/3 1 1. 5 2
0 1 2 3 5
1 2 3 7,5 10
2 3 7,5 15/10 25
3 25/30 50
4 40/50 100
5 75/100 200

РЕЙТИНГ NEMA ДЛЯ КОРПУСОВ

NEMA и другие организации учредили
стандарты конструкции корпуса для управляющего оборудования.В общем,
оборудование будет закрыто по одной или нескольким из следующих причин:

  1. Не допускайте случайного контакта с токоведущими частями.
  2. Защитите контроль от вредного воздействия окружающей среды
    условия.
  3. Предотвратить взрыв или возгорание, которые могут
    возникает из-за электрической дуги, вызванной регулятором.

Общие типы корпусов по классификации NEMA
числа:

NEMA I — ОБЩЕГО НАЗНАЧЕНИЯ

Корпус общего назначения предназначен
в первую очередь для предотвращения случайного контакта с закрытым аппаратом.Это
подходит для общего применения внутри помещений, где он не подвергается воздействию
необычным условиям эксплуатации. Корпус NEMA I служит защитой
от пыли и легких непрямых брызг, но не пыленепроницаемый.

NEMA 3 — DUSTTIGHT, RAINTIGHT

Этот корпус предназначен для
подходящая защита от указанных погодных опасностей. Корпус NEMA 3
подходит для применения на открытом воздухе, на корабельных доках, каналах и строительстве
работа, и для применения в метро и туннелях.Он также устойчив к мокрому снегу.

NEMA 3R — ДОЖДЕВОЙ, УСТОЙЧИВЫЙ НА СЛОИ

Этот корпус защищает от помех
в работе встроенного оборудования из-за дождя и устойчиво к повреждениям
от воздействия мокрого снега. Он разработан со ступицами для кабелепровода и внешним креплением,
а также дренажные положения.

NEMA 4 — ВОДА

Водонепроницаемый корпус разработан
чтобы выдержать испытание шланга, описанное в следующем примечании: «Корпуса должны
быть испытанным погружением в струю воды.Шланг с однодюймовым соплом
должны использоваться и должны обеспечивать не менее 65 галлонов в минуту. Вода
должно быть направлено на ограждение с расстояния не менее 10
стопы и в течение пяти минут. В этот период может быть направлено
в любом одном или нескольких направлениях по желанию. Не должно быть утечки
вода в корпус в этих условиях ».

Корпус NEMA 4 подходит для
применение на открытом воздухе в доках судов и на молочных заводах, пивоварнях и т. д.

NEMA 4X — ВОДОНЕПРОНИЦАЕМЫЙ, КОРРОЗИОННОСТОЙКИЙ

Эти корпуса обычно
в соответствии с корпусами NEMA 4, за исключением того, что они сделаны из материала
что очень устойчиво к коррозии.По этой причине они идеальны
в таких областях, как бумажные фабрики, упаковка мяса, удобрения и химикаты
заводы, на которых загрязняющие вещества обычно разрушают стальную ограду над
Период времени.

NEMA 7 — ОПАСНЫЕ ЗОНА — КЛАСС
I

Эти корпуса предназначены для
требования к применению Национального электрического кодекса для класса
Я опасные места. В этом типе оборудования прерывание цепи
происходит в воздухе.

«Объектами класса I являются те, в которых
горючие газы или пары присутствуют или могут присутствовать в воздухе в количествах
достаточно для образования взрывоопасных или горючих смесей.»

NEMA 9 ОПАСНЫХ МЕСТ — КЛАСС
II

Эти корпуса предназначены для
требования к применению Национального электрического кодекса для класса
II опасные места.

«Объекты класса II:
опасны из-за наличия горючей пыли ».

Буква или буквы после
номер типа указывает на конкретную группу или группы опасных мест
(как определено в Национальном электротехническом кодексе), для которого
разработан.Обозначение неполное без буквы или букв суффикса.

NEMA 12 — ПРОМЫШЛЕННОЕ ПРИМЕНЕНИЕ

Корпус NEMA 12 предназначен для
использование в тех отраслях, где желательно исключить такие материалы, как
пыль, ворс, волокна и муха, масло см. страницу или охлаждающую жидкость см. страницу. Там
в корпусе отсутствуют отверстия для кабелепровода или выбоины, а монтаж
с помощью фланцев или монтажных ножек.

NEMA 13 — МАСЛЯНЫЙ, DUSTTIGHT

Корпуса NEMA 13 обычно
Литая конструкция, разборная, что позволяет использовать в тех же условиях, что и NEMA
12 устройств.Существенное отличие состоит в том, что благодаря литому корпусу
вход для кабелепровода является неотъемлемой частью корпуса NEMA 13,
и установка осуществляется с помощью глухих отверстий, а не монтажных скоб.

Резисторы в схемах — Практика — Физический гипертекст

Начнем процесс с объединения резисторов. В этой схеме четыре последовательных пары.

слева
R с = 3 Ом + 1 Ом
R с = 4 Ом
R с = 4 Ом + 2 Ом
R с = 6 Ом
правый
R с = 2 Ом + 3 Ом
R с = 5 Ом
R с = 1 Ом + 4 Ом
R с = 5 Ом

Эти пары образуют две параллельные цепи, одну слева и одну справа.

слева
1 = 1 + 1
R p 4 Ом 6 Ом
R p = 12 Ом = 2.4 Ом
5
правый
1 = 1 + 1
R p 5 Ом 5 Ом
R p = 5 Ом = 2. 5 Ом
2

Каждый набор из четырех резисторов включен последовательно с другим.

слева
R с = 2,4 Ом + 0,6 Ом
R с = 3 Ом
правый
R с = 2,5 Ом + 0,5 Ом
R с = 3 Ом

Левая и правая половины цепи параллельны друг другу и батарее.

1 = 1 + 1 = 2
R p 3 Ом 3 Ом 3 Ом
R p = 3 Ом = 1. 5 Ом
2

Теперь, когда у нас есть эффективное сопротивление всей цепи, давайте определим ток от источника питания, используя закон Ома.

I итого = V всего + 24 В = 16 А
R Итого 1.5 Ом

А теперь пройдемся по цепи (не буквально, конечно). На каждом соединении ток будет делиться: больше по пути с меньшим сопротивлением и меньше по пути с большим сопротивлением. Поскольку заряд не протекает нигде в полной цепи, ток будет одинаковым для всех элементов, последовательно соединенных друг с другом.

Левая и правая половины схемы идентичны по общему сопротивлению, что означает, что ток будет равномерно делиться между ними.

8 A для резистора 0,6 Ом
на слева .
8 A для резистора 0,5 Ом
на правой стороне .

С каждой стороны ток снова делится на две параллельные ветви.

Ветви слева имеют сопротивления в соотношении…
R 1 и 3 = 4 Ом + 2
R 2 и 4 6 Ом 3
то есть токи разделятся в соотношении…
для резисторов 1 Ом и 3 Ом
на слева .

Добавить комментарий

Ваш адрес email не будет опубликован. Обязательные поля помечены *